SlideShare a Scribd company logo
1 of 71
Parkinson’s disease
Discussion by ShivankanKakkar, MD
Long Case
Day 1
Mr LN, a 67-year-old retired IAS Officer, was admitted to the
neurology ward. His presenting complaints included difficulty in
getting up from chairs and initiating walking. Eighteen months earlier
he had noticed that his self-winding watch, which he wore on his
right wrist, was consistently losing time. When moved to his left
wrist, the watch kept perfect time. Friends had pointed out that he
had developed a limp, dragging his right foot. These symptoms had
worsened over the previous 18 months, and he had also developed
a resting tremor in his right hand and leg. He had noticed changes
in his bowel habits many years previously. His general practitioner
(GP) suspected Parkinson’s disease and referred Mr LN to a
neurologist for confirmation of the diagnosis.
Q1 What are the main symptoms of Parkinson’s disease?
Q2 Is it possible to make a definitive diagnosis of idiopathic Parkinson’s
disease (IPD)?
Q3 What biochemical defects are thought to be present in a patient such
as Mr LN?
Q4 Outline a treatment care plan for Mr LN.
Q5 Which drugs are usually considered for the initial treatment of
patients with IPD and what are their modes of action?
Day 2 Mr LN was started onropinirole.
Q6 Do you agree with this choice?
Q7 How should Mr LN’s ropinirole therapy be adjusted to optimise
his response?
Q8 What counselling points would you highlight to a patient
commencing ropinirole?
Day 13 Mr LN was discharged on ropinirole 500 micrograms three times
daily and instructed to titrate the dose as directed. Mr LN’s GP was sent
a letter describing his recent admission and advice on how to titrate the
dose of ropinirole further.
Month 18 Mr LN was admitted for reassessment. Despite never
completely abating, Mr LN’s symptoms of bradykinesia and rigidity
had temporarily improved following initiation and up-titration of
ropinirole; however, recently Mr LN had experienced a worsening of
these symptoms, and his wife reported that he had become rather
obsessive about cleaning out the garden shed, performing this task
on an almost daily basis. On admission he was taking ropinirole 6 mg
three times daily.
Q9 What recommendations would you make with regard to Mr LN’s
drug therapy?
It was decided to start Mr LN on Sinemet Plus 125 mg three times
daily and simultaneously to reduce the dose of his dopamine agonist.
Q10 How can the adverse effects of levodopa be minimised?
Month 18, Day 8 Mr LN was discharged. He was now stabilised on
Sinemet Plus, two tablets three times daily, and ropinirole 4 mg
three times daily and had experienced a significant improvement in
his motor function.
Month 66 Mr LN was again admitted for reassessment. His therapy
and symptoms had remained stable on the Sinemet and ropinirole
until recently, when he had started to notice his arms shaking about 1
hour after each dose of Sinemet Plus. This lasted for about 45
minutes before stopping. Mr LN also complained of painful dystonic
cramps at night, which caused him to wake early most mornings. He
had attempted to counter this by taking an extra Sinemet Plus tablet
shortly before going to bed, which sometimes helped. His wife added
that he was becoming profoundly immobile up to 2 hours before his
Sinemet doses were due during the day. On examination he was
found to have a mask-like face and a resting tremor in both hands
and legs. ‘Cogwheel’ rigidity was found in all four limbs. He had
difficulty initiating speech and his voice was very quiet. He struggled
to rise from his chair, and had a characteristic ‘parkinsonian shuffle’
when walking. When his shoulders were pulled forward from
standing he staggered forward and had to be supported to stop him
falling.
Mr LN complained of occasional falls and was not confident to
leave the house. He relied on his wife to do everything for him. He
admitted to being depressed about his condition.
There was no other medical history of note. All routine laboratory
tests were within normal ranges. His drug therapy was ropinirole 4
mg three times daily plus two Sinemet Plus tablets at 8 a.m., 2 p.m.
and 7 p.m. and, occasionally, one Sinemet Plus tablet at11 p.m.
Q11 Which of the long-term complications of levodopa therapy is
Mr LN suffering from?
Q12 What alterations to Mr LN’s drug therapy would you recommend
in order to try to minimise these effects?
Q13 How could you monitor and assess Mr LN’s response to these changes?
Q14 Can you suggest any non-drug management that might benefit Mr LN?
Mr LN’s drug therapy was changed to one Madopar dispersible
62.5 mgtablet on waking around 7 a.m., and one Sinemet Plus tablet
at 10 a.m., 1 p.m., 4 p.m. and 7 p.m. In addition, he was prescribed
one Sinemet CR tablet at 10 p.m. The nursing staff were asked to
complete an hourly ‘on– off’ chart.
Month 66, Day 6 Mr LN’s mobility and dyskinesias were improved;
however, he remained depressed about his condition.
Q15 Would Mr LN benefit from an antidepressant?
Q16 If so, which would you choose?
Completed on/off chart
Time Day 1 Day 5
7 a.m. off off (dose)
8 a.m. off (dose) on (dyskinesia)
9 a.m. on on
(dyskinesia)
10 a.m. on off (dose)
(dyskinesia)
11 a.m. on on (dyskinesia)
12 noon off on
1 p.m. off on (dose)
2 p.m. off (dose) on (dyskinesia)
3 p.m. on on
(dyskinesia)
Month 66, Day 8 Mr LN had noticed a significant reduction in the
shaking of his arms following his recent regimen change; however,
he was still becoming considerably immobile about 1 hour before his
dose of Sinemet. His sleep had improved so that he was getting at
least 6 continuous hours’ sleep and no longer woke in pain from
cramps. The consultant neurologist recommended commencing
entacapone.
Q17 Why has the consultant neurologist recommended entacapone,
and does this necessitate the adjustment of Mr LN’s other IPD
therapy?
Q18 What counselling points would you highlight to a patient
commencing entacapone?
Month 66, Day 11 During the ward round, you noted that Mr LN’s
morning medication was still on his bedside table. His medication
now comprised one Madopar dispersible 62.5 mg tablet on waking
around 7 a.m., one Sinemet Plus tablet at 10 a.m., 1 p.m., 4 p.m.
and 7 p.m. and a Sinemet CR tablet at 10 p.m. In addition, he was
taking one entacapone tablet alongside each dose of Sinemet Plus
and 3 mg ropinirole three times a day (made up of a 1 mg and a 2 mg
tablet for each dose). His regimen thus comprised a total of 16
tablets per day.
On questioning, Mr LN informed you that, although he was feeling
better, he was concerned at the number of tablets he was taking and
feared that he would have problems remembering to take them all
when he returned home. He also found the new tablet quite hard to
swallow because of its size.
Q19 In view of Mr LN’s concerns, what options are there for
rationalising his medications?
Month 75 Mr LN was admitted as an emergency by his GP, having
developed visual and auditory hallucinations over the previous week.
He was hearing voices talking about him, threatening to kill him. He
was also seeing insects crawling up the walls and burrowing into his
skin. On examination he was clearly distressed and very frightened.
His medication on admission was one Madopar dispersible tablet on
waking, one Stalevo 100/25/200 tablet four times daily, one Sinemet
CR tablet at night and oneropinirole XL tablet 8 mg daily.
Q20 Which drugs might have contributed to Mr LN’s symptoms? Q21
What adjustments would you recommend be made to Mr LN’s
medication?
Month 75, Day 7 The recommendations had been carried out.
Mr LN’s visual hallucinations had improved, but he was still
experiencing
distressing auditory hallucinations and the control of his symptoms had
deteriorated, such that he was experiencing frequent ‘off’ periods.
Q22 What course of action would you suggest to improve Mr LN’s symptoms?
Mr LN was started on rivastigmine 1.5 mg twice daily. The dose
was increased over the next 10 days to 4.5 mg twice daily. His
dopaminergic therapy was adjusted to one Stalevo 100/25/200
tablet five times daily, one Sinemet CR tablet at night and one
Madopar 62.5 mg dispersible tablet upon waking. His ropinirole XL
therapy was discontinued. His hallucinations resolved and acceptable
control of his Parkinson’s symptoms was achieved. He was
discharged on this regimen.
Q23 What is the long-term outlook for Mr LN?
The main symptoms in patients with Parkinson’s disease are tremor,
rigidity, bradykinesia (slowness of movement), akinesia (loss of
movement) and postural abnormalities.
What are the main symptoms of Parkinson’s disease?
A1
The onset of Parkinson’s disease is usually insidious and progression
slow. Many patients first notice a resting tremor. This usuallyinitially
affects the hands and may be unilateral. The tremor disappears on
movement and during sleep, and may be worse under stress. The
patient is usually over 50 years old on presentation.
Rigidity manifests as an increased resistance to passive movement
and is classically termed ‘cogwheel’ rigidity, with a ratchet-like phe-
nomenon felt at the wrist on passive movement of the hand.
Bradykinesia manifests as a general slowness in movement.
Together with the rigidity, it is responsible for the typical
abnormalities of gait: difficulty in starting and finishing steps,
resulting in shuffling; a stooped head; flexed neck, upper extremities
and knees; and a lack of normal arm swing. A wide range of non-
motor symptoms (NMS) have also been described in Parkinson’s
disease, all of which can have a significant impact on quality of life.
These include bowel and bladder problems, fatigue, pain, sleep
disorders and autonomic dysfunction, in addition to difficulty in
swallowing and speech alteration, drooling of saliva and olfactory
disturbance. Loss of postural reflexes leads to postural imbalance and
sometimes to frequent falls. NMS are common and can occur at all
stages of Parkinson’s disease, including long beforediagnosis.
Symptoms increase in number and severity as the disease progresses.
Until recently, there has been a tendency for healthcare professionals
to overlook these features in favour of the more apparent motor
symptoms.
Is it possible to make a definitive diagnosis of idiopathic
Parkinson’s disease (IPD)?
A2 At present the only way of making a definite diagnosis of IPD
is by postmortem study of the brain.
IPD is the most common cause of parkinsonism, accounting for
approxiimately 75% of cases presenting to neurologists. Other causes
include other neurodegenerative diseases such as progressive
supranuclear palsy (PSP) and multiple system atrophy (MSA),
intoxication with heavy metals, treatment with therapeutic drugs
(neuroleptics, metoclopramide) and chronic cerebrovascular disease.
The definitive diagnosis of Parkinson’s disease is based on
characteristic neuropathological findings of Lewy bodies and
neuronal loss in the substantia nigra and other brainstem nuclei.
Studies have shown that only 65– 75% of patients diagnosed as
having early Parkinson’s disease had the characteristic findings at
postmortem.
Current opinion and guidelines recommend that all patients with
a ‘suspected’ diagnosis of IPD must be referred untreated to a
specialist who can reliably differentiate between IPD and other
parkinsonian syndromes. Previously the response to a ‘challenge’ of
levodopa or dopaminergic agents has been used for diagnostic
purposes; however the National Institute for Health and Clinical
Excellence (NICE) guidance for Parkinson’s disease does not
advocate that this be performed routinely, although guidelines vary
worldwide. Acute challenge testing is similar to, but not better than,
chronic levodopa therapy in terms of diagnostic accuracy, but is
associated with adverse effects and extra costs.
The use of imaging techniques is increasing in this field. Single
photon emission computed tomography (SPECT) with DatSCAN, a
radi- olabelled cocaine derivative, can be used to measure the
amount of dopamine-releasing neurones in the brain. This type of
imaging can aid differentiation between parkinsonian and non-
parkinsonian syndromes, but it is unable to distinguish between IPD,
MSA and PSP.
What biochemical defects are thought to be present in a patient
A3
such as Mr LN?
Several biochemical defects are thought to be present in
patients with Parkinson’s disease.
A combination of cholinergic (excitatory) and dopaminergic
(inhibitory) mechanisms acting in the striatal tracts of the basal
ganglia of the brain are thought to be responsible for the smooth
control of voluntary movements.
Imbalances in the neurotransmitters lead to movement disorders. In
patients with Parkinson’s disease, dopamine concentrations in the
three major parts of the basal ganglia are reduced to a fraction of
normal. Compensatory mechanisms operate and symptoms are not
noted until a severe loss (80%) of dopaminergic neurones has
occurred. The severity of some symptoms, such as bradykinesia, has
been found to correlate with striatal dopamine levels; however,
abnormalities of other neurotrans- mitters, including norepinephrine
(noradrenaline), 5-hydroxytryptamine (serotonin) and gamma-
aminobutyric acid, have also been reported. The full relevance of
these changes is unclear.
Outline a treatment care plan for Mr LN.
A4 Thegoals of symptomatic drug treatment are to help the patient
function independently for as long as possible, and to achieve
this with the minimum of adverse effects.
Patients with Parkinson’s disease have a chronic deteriorating condition
which will result in lifelong drug therapy of increasing complexity.
A long-term treatment care plan would include:
(a) Involving Mr LN in the choice of appropriate initial therapy.
(b) Ensuring the development of appropriate treatment outcome
mea- sures and a suitable treatment-monitoring programme.
(c) Ensuring that the patient understands the role of drugs inthe
symptomatic treatment of the disease and their possible adverse
effects.
(d) Ensuring that the patient and carers understand the importance
of adherence and timing of drug doses.
(e) Anticipating problems such as the potential for nausea and vomiting
with levodopa preparations, and offering appropriate advice on their
prevention.
(f) Counselling the patient and carers about drugs that should be
avoided in Parkinson’s disease. These include medicines which act
as dopamine antagonists, such as metoclopramide and the older
antipsychotics (e.g. chlorpromazine, haloperidol).
(g) As the disease progresses and more drugs are added to the
regimen, medicine taking may become problematic and advice on
methods for improving and maintaining adequate adherence
should be given.
(h) As most patients with IPD will be elderly, the general considerations
given to elderly patients should also be applied.
As treatment may continue for many years and become increasingly
complex, continuity of pharmaceutical care is an issue and
consideration should be given to a personal, individualised patient
record that can be used by pharmacists at various stages of the
disease. The Non-pharmacological considerations include education on
the disease itself, advice on diet, exercise programmes, and other areas
such as driving, alcohol intake and recreational activities.
Which drugs are usually considered for the initial treatment of
patients with IPD and what are their modes of action?
A5 Initial drug therapy as recommended by NICE guidelines forboth
early- and late-onset IPD will usually be chosen from the
following: levodopa preparations; a dopamine agonist; or a
monoamine oxidase B inhibitor (MAOBI).
There is still considerable debate about the best initial choice of
therapy for patients with IPD, and also when to start symptomatic
treatment. Most experts now advocate early treatment to provide
patients with maximal clinical benefit at the start of their illness.
Most Parkinson’s disease specialists start treatment when a patient’s
symptoms begin to interfere with their lifestyle. What constitutes
this will vary from patient to patient, but may include impairment of
activities of daily living (ADL), threatened loss of employment or gait
disturbance with a risk of falling.
When a decision is made to initiate treatment, the age of the
patient, the degree and type of symptoms and the patient’s
expectations will influence drug choice. Patients should be made
aware that drug treatment can provide symptomatic relief, but that
there is at present no way of halting the progression of the disease.
Levodopa is the most effective drug in the symptomatic management
of Parkinson’s disease, and virtually all patients will experience
meaningful benefit; however, it can cause significant short- and long-
term adverse effects.
The rationale for early use of dopamine agonists is that they
provide some benefit in early disease, but are significantly less likely
to lead to the development of motor complications, particularly
dyskinesias. Recent studies comparing levodopa with pramipexole in
early disease have shown that levodopa more effectively manages
motor symptoms, but is related to a greater incidence of motor
complications. Treatment withdrawal through adverse effects (e.g.
somnolence and peripheral oedema) was higher in the pramipexole
group. The incidence of impulse control disorders with dopamine
agonists is reported to be around 17% and can have significant
impact. The once daily formulations of pramipexole and ropinirole
have an advantage not only in compliance, but also in managing
symptoms for young patients who are still working, and who donot
wish for the higher frequency of dosing required with levodopa.
While it is an individual decision between the patient and the
consultant, initiation of a dopamine agonist in patients who require
this but who still have relatively mild symptoms may be appropriate.
Patients with more severe symptoms and those over 70 (in whom the
development of motor complications is less likely) should probably
be started on a levodopa preparation. Anticholinergic drugs are now
rarely used for the treatment of Parkinson’s disease. They provide some
relief of tremor but are of little value in the treatment of other features,
such as rigidity and bradykinesia. In addition, adverse effects are
common. These include peripheral effects such as dry mouth, blurred
vision and constipation, as well as potentially serious central effects,
including confusion and hallucinations. Their use is best reserved for
younger patients (under 60), in whom resting tremor is the
predominant feature. Anticholinergic drugs are thought to act by
correcting the relative central cholinergic excess brought about by
dopamine deficiency. They are not recommended as first-line treatment.
The MAOBIs selegiline and rasagiline selectively inhibit monoamine
oxidase B, one of the enzyme systems which break down dopamine.
The action of endogenous dopamine is thus augmented.
Selegiline has a mild antiparkinsonian effect and is sometimes used
either alone in early disease, or later to potentiate the action of
levodopa preparations. Selegi- line is metabolised to amfetamine
derivatives, so it may have an alerting effect, especially at night,
when it can cause insomnia, vivid dreams andnightmares.
Rasagiline, although very similar in chemical structure, isnot
metabolised to amfetamine derivatives and so is potentially free of
alerting side-effects. MAOBIs have been investigated for possession
of neuroprotective properties, but this has not yet been proven.
Dopamine deficiency cannot be rectified by the administration of
dopamine, because dopamine does not cross the blood– brain barrier.
Lev- odopa does cross the blood– brain barrier and is converted to
dopamine in the basal ganglia. Levodopa is thus thought to act primarily
by increasing brain dopamine concentrations. If levodopa is
administered alone, over 95% of a dose is decarboxylated to dopamine
peripherally, which results in reduced amounts being available to cross
the blood– brain barrier and problematic peripheral side-effects such as
nausea, vomiting and postural hypotension. Levodopa is therefore now
almost always administered with a dopa-decarboxylase inhibitor,
either carbidopa or benserazide.
These dopa-decarboxylase inhibitors do not cross the blood– brain
barrier and so smaller daily doses of levodopa can be administered,
thereby reducing the incidence of peripheral side-effects. Levodopa
therapy is particularly helpful in controlling symptoms of bradykinesia
or akinesia.
Do you agree with this choice?
A6 Yes. The choice of initial therapy is based on many different
factors that will vary from patient to patient. The useof a
dopamine agonist is indeed appropriate in this situation, as
would be the use of levodopa.
The dopamine agonists can be split into two different categories: ergot
and non-ergot. Ergot-derived dopamine agonists include the older-
generation products bromocriptine, lisuride, pergolide and cabergoline.
Non-ergot- derived agonists include the newer agents pramipexole,
ropinirole and rotigotine.
Use of the ergot-derived agonists has declined considerably over
the last few years as a result of increased concerns about long-term
side- effects, specifically cardiac fibrosis. There are clear monitoring
guidelines for patients who remain on these agents.
Critical appraisal of the literature evaluating the two non-ergot-
derived oral agonists provides no evidence to prefer one over the other.
Rotigotine, the transdermal patch formulation, although considered less
potent than the other non-ergot-derived dopamine agonists, provides
con- tinuous dopaminergic stimulation (CDS) over a 24-hour period.
Although it is suggested that 24-hour continuous drug release more
closely resembles endogenous dopamine release and may reduce the
risk of motor fluctu- ations and dyskinesias developing, there are at
present no firm clinical data to support this. Ropinirole XL and
pramipexole PR are also available as once-daily preparations. The initial
requirement to stabilise patients with immediate-release ropinirole prior
to starting once-daily (XL) has now been removed, and where local
formulary allows, prolonged-release formulations can be used from
initiation.
How should Mr LN’s ropinirole therapy be adjusted to optimise
his response?
A7 Like any dopaminergic therapy, ropinirole must be started ata
low dose and increased gradually in order to reduce the
incidence of side-effects and to establish the lowest effective
dose.
In order to facilitate gradual up-titration, ropinirole is available as a
‘starter pack’ that contains all the medication required for a stepwise
dose titration over a 4-week period. A ‘follow-on’ pack allows further
stepwise titration
over a similar period to a therapeutic dose. This dose can then be
further increased as required and tolerated by the patient.
Rotigotine patches are also available in a starter pack format.
Pramipexole doses are often referred to in both the salt and the base
form. It is important to be aware of this when referring to doses with
patients and other healthcare professionals; there are a number of
information materials available to aid patients and reduce confusion.
What counselling points would you highlight to apatient
A8
commencing ropinirole?
Patients should be warned about the potential for dopamine
agonists to cause impulse control disorders. In addition they
should be counselled on the risk of excessive daytime
somnolence and the implications of this on driving/operating
machinery.
What recommendations would you make with regard to Mr LN’s
A9
drug therapy?
Mr LN should at this stage be commenced on a low dose of
levodopa as an adjunct to the dopamine agonist. The dopamine
agonist dose should be reduced in view of his history of pund-
ing (the obsessive and repetitive perfomance of a useless task).
The levodopa dose should be gradually increased, with decisions
based on symptom relief and adverse effects.
Levodopa and dopa-decarboxylase inhibitors are marketed in the UK as
Sinemet (levodopa plus carbidopa: co-careldopa) and Madopar
(levodopa plus benserazide: co-beneldopa). There are six Madopar
preparations and six Sinemet preparations available in a variety of
levodopa strengths and formulations (standard, dispersible and
controlled release). The potential for confusion among clinicians and
pharmacists is considerable, and care must be taken to ensure that the
patient receives the intended preparation. At the start of levodopa
therapy the effects of a dose usually last for
4– 8 hours, so the tablets may be prescribed three times daily. The dose
should be increased by one tablet every 2– 3 days until optimum effects
are seen or adverse effects occur. If daily doses of <70 mg carbidopa are
used, the peripheral decarboxylase will not be saturated and Mr LN will
be more likely to suffer from nausea and vomiting. For this reason there
are low-levodopa Sinemet preparations containing 25 mg carbidopa plus
100 mg levodopa. There are also preparations containing 10 mg
carbidopa plus 100 mg levodopa that are suitable when higher levodopa
doses are needed. This is not a problem with benserazide preparations.
Most patients are initially controlled on 400– 800 mg levodopa daily.
Mr LN is beginning to display symptomsof punding. This is
charac- terised by repetitive pointless behaviours carried out for long
periods of time at the expense of all other activities, and is associated
with excessive dopaminergic stimulation. Punding and impulse
control disorders, for example compulsive gambling, can be
devastating and occur in up to 17% of patients treated with
dopamine agonists. The risk is higher in younger patients with a
history of addiction.
Punding is managed by reducing and discontinuing dopaminergic
stimulatory agents, predominantly dopamine agonists, although
levodopa is also implicated. In view of Mr LN’s worsening motor
function it is reasonable to reduce his dopamineagonist to manage
the side-effect of punding, and to supplement this with (initially) low
doses of levodopa in view of its superiority in improving motor
function. Abrupt withdrawal of dopamine agonists has been
associated with neuroleptic malignant syndrome and should
therefore be avoided. As there are no defined reduction regimens for
these agents, the rate of withdrawal can vary. In the case of Mr LN,
complete withdrawal may not be necessary and so slow reduction
would be appropriate, depending on symptom control. Reducing the
daily dose by 3 mg at 5-day intervals would be a reasonable regimen.
How can the adverse effects of levodopa be minimised?
A10 The peripheral side-effects of levodopa are significantly
reduced by combining it with a dopa-decarboxylase inhibitor
(see A5); however, peripheral side-effects may still occur.
Levodopa therapy is therefore best started at a low dose and
increased gradually. In addition, ensuring that the drug is taken with
or after food can reduce the incidence of nausea and vomiting still
further. Using domperidone, a dopamine antagonist that does not
cross the blood– brain barrier, can also reduce nausea and vomiting.
Doses of 10– 20 mg domperidone 1 hour before levodopa
preparations areeffective.
Which of the long-term complications of levodopa therapy is
Mr LN suffering from?
A11 End-of-dose akinesia and dyskinesias.
Initial treatment with levodopa leads to sustained improvement
through- out the day. Most patients will show a slow improvement in
response during the first 18– 24 months of treatment. Symptoms are
then ade- quately controlled for 3– 5 years. Unfortunately, after
long-term treat- ment (>5 years) only about 25% of patients continue
to have a good, smooth response. The main complications that
develop are fluctuations
in response, dyskinesias, psychiatric side-effects and partial or
substantial loss of efficacy.
Fluctuations in response initially consist of end-of-dose deteriora-
tion or end-of-dose akinesia. End-of-dose akinesia is the term used
when the therapeutic effects of a dose of levodopa are lost. This
commonly occurs first thing in the morning (after the longest dosage
interval) or just before or after a dose during the day, when the effect of
the previous dose wears off and before the next one is taken or
becomes effective. Patients may become immobilised and unable to do
anything except wait for the next dose. After prolonged treatment,
gradual deterioration in symptoms may begin between 1 and 3 hours
after a dose.
Long-term levodopa therapy is also associated with the ‘on– off’
phenomenon. The patient develops a sudden loss of effectiveness (off),
when ‘freezing’ occurs, which may last for only a minute or for up to
several hours before normal function returns (on).
Dyskinesias (abnormal movements) can occur in 60– 90% of patients
and are usually dose-related. They are generally worse when the
response to a dose is maximal (peak dose), and have been correlated
with high levodopa plasma levels. They are therefore commonly seen
after a dose has been taken. Symptoms include grimacing, gnawing and
involuntary rhythmic jerking movements.
What alterations to Mr LN’s therapy would you recommend in
order to try to minimise these effects?
A12 Increase the frequency of levodopa administration to 3-
hourly and reduce the amount given with each dose.
The most common approach to response fluctuation is to reduce each
individual dose of levodopa and to increase the frequency of
administration. This can reduce end-of-dose akinesia, but care must be
taken to ensure a clinical response is maintained.
End-of-dose bradykinesia or akinesia is thought to be caused by a
progression of the underlying disease or an unexplained occurrence
of symptoms of dopamine deficiency after an initial response to each
dose. Although there is no change in the plasma half-life of levodopa,
it appears that the pharmacological half-life is reduced. End-of-dose
akinesia has been shown to be corrected by levodopa infusion;
however, this is not a genuine therapeutic option, as levodopa must
be infused in large volumes, owing to its acidity, and it also
commonly causes thrombophlebitis. The initial approach taken to
minimise the end-of-dose effect is therefore to try to reduce the
dosage interval.
Modified-release versions of co-careldopa (Sinemet) and co-
beneldopa (Madopar) are available which result in a more prolonged
and
constant plasma level of levodopa. Both preparations lead to delayed
action and lower peak concentrations than standard levodopa, but
the risk of dose failure may be higher. Patients may still need to take
controlled-release preparations every 3 or 4 hours, and may need doses
of standard preparations to produce optimal clinical effects, especially
with the first dose of the day.
Dyskinesias may occur at the time of peak benefit, at the beginning
or end of a dose, or both (diphasic), or during ‘off’ periods. A reduction
in levodopa dose may reduce peak dose dyskinesias, as may the partial
replacement of levodopa with a dopamine agonist. Diphasic
dyskinesias may also be helped by partially replacing levodopa with
dopamine agonists. ‘Off’ period dyskinesia may be improved by
administering a fast- acting (e.g. dispersible) preparation or, for those
occurring in the early morning, a controlled-release preparation last
thing at night.
Other measures to improve consistency of absorption should also
be considered. This includes treating constipation (a common
problem in Parkinson’s) and reviewing medications which can slow
gastric emptying. Taking doses 1 hour before food (if tolerated) can
also be helpful. High- protein meals may also reducelevodopa
bioavailability because large neutral amino acids can compete with
levodopa for absorption.
How could you monitor and assess Mr LN’s response to these
A13
changes?
By charting his mobility regularly.
Some neurology wards use mobility charts on which an indication of
a patient’s mobility can be recorded at suitable intervals. Such charts
can be a valuable aid to the manipulation of drug administration in
order to optimise therapy. In addition to a score, a brief description
of the patient’s condition may be added.
Can you suggest any non-drug management that might benefitMr
A14
LN?
Mr LN may benefit from some remedial therapy, such as speech
and language therapy, physiotherapy or occupational therapy.
This should beavailable to patients with IPD and has been
endorsed in NICE guidance for IPD.
The role of these therapies is to maintain the maximum level of
functional mobility and capacity to perform ADL. Early therapeutic
intervention cannot reverse the course of Parkinson’s disease, but it
can delay potential deformity and functional decline.
Physiotherapy for Parkinson’s disease patients addresses the functional limitations
caused by rigidity and bradykinesia. It may include an
On No rigidity, mobilising
Off Rigid, with or without
tremor
Unable to mobilise or only
with assistance
On with dyskinesia No rigidity, mobilising but
with involuntary
movements
State Description
exercise programme that focuses on maintaining flexibility,
balance and strength.
Scoring system for patients with PD chart
Would Mr LN benefit from an antidepressant?
A15 Probably.
Depression is very common in Parkinson’s disease; approximately 40–
50% of patients suffer from depression at least once during the
course of their disease. Depression in Parkinson’s disease is
characterised by feelings of guilt, helplessness, remorse and sadness.
It is independent of age, disease duration, severity of symptoms or
cognitive impairment.
Ensuring adequate treatment for Parkinson’s disease should be the
first step before considering more specific antidepressant therapy. This
has been achieved in Mr LN, so a trial of an antidepressant would be
reasonable.
If so, which would you choose?
A16 A tricyclic antidepressant (TCA) or selective serotonin
reuptake inhibitor (SSRI) could be prescribed for Mr LN.
At present, there is insufficient evidence to recommend one antidepres-
sant/antidepressant class over another. This lack of data is also
highlighted in the NICE guidance. Clinical practice, as well as trial data,
supports the use of some TCAs (e.g. amitriptyline, nortriptyline) in
Parkinson’s dis- ease; however, they are often associated with
anticholinergic effects and orthostatic hypotension, which may limit
their usefulness. The SSRIs (e.g. fluoxetine, sertraline, citalopram)
have also been shown to be effective in Parkinson’s disease. They are
free of the anti- cholinergic effects associated with the tricyclics, and
theoretical concerns that they may worsen parkinsonian symptoms
have not been borne out by recent studies.
Why has the consultant neurologist recommended entacapone,
and does this necessitate the adjustment of Mr LN’s other IPD
therapy?
A17 Mr LN is still displaying signs of end-of-dose akinesia, despite
appropriate adjustment to his levodopa therapy. Entacapone
ther- apy may improve these symptoms, but his otherIPD
therapies will need to be adjusted as the drug is introduced.
Entacapone is a catechol-O-methyltransferase (COMT) inhibitor and
works in synergy with Sinemet and Madopar preparations, resulting in a
30– 50% increase in levodopa half-life. Entacapone works by inhibiting
the peripheral metabolism of levodopa by the enzyme COMT, thereby
increasing its availability to the brain. Entacapone is licensed for use as
an adjunct to levodopa in Parkinson’s disease. Studies have shown that
its use can significantly reduce ‘off’ time and increase ‘on’ time in
patients with ‘wearing-off’ episodes. It should be given at a dose of 200
mg with each dose of levodopa, up to 10 times daily. The introduction
of entacapone to a regimen can cause a new onset or worsening of
existing dyskinesias. In order to minimise this, the daily dose of
levodopa should be reduced by about 10– 30% by extending
the dosing intervals and/or by reducing the amount of levodopa per
dose. In practice, the choice varies from patient to patient.
Tolcapone, the only other commercially available COMT inhibitor,
differs from entacapone in that it also blocks centralCOMT.
Tolcapone was withdrawn in 1998 after a number of cases of
hepatitis, but has since been relaunched with strict monitoring
parameters and guidance onuse.
What counselling points would you highlight to a patient
A18
commencing entacapone?
Entacapone can discolour the urine reddish brown. Patients
should also be advised that nausea and vomiting can occur due
to augmentation of levodopa, and that diarrhoea is a common
adverse effect.
The large size of the tablets maycause problems for patients with
swallowing difficulties.
In view of Mr LN’s concerns, what options are there for
A19
rationalising his medications?
The use of combination products and alternative
formulations could be considered.
In the literature it has been noted that approximately 20% of
patients with IPD are non-adherent with their medication. Younger
patients and those with complex regimens are associated with the
highest levels of non-adherence.
A number of pharmaceutical companies have recently developed
combined preparations and modified-release preparations in an
attempt to reduce the patient’s ‘pillburden’.
Stalevo is a combined preparation of levodopa, carbidopa and
entacapone formulated into one tablet. A Stalevo tablet is smaller
than a tablet of entacapone alone andis thus also helpful in patients
with swallowing difficulties who require therapy. Stalevo is available
in a number of strengths, although dose titration is more limited
than with the individual components. Mr LN could be started on
Stalevo in place of Sinemet and entacapone.
The transdermal patch rotigotine and the oral formulations
ropini- role XL and pramipexole PR are designed for once-daily
administration. Rotigotine is particularly useful where swallowing
difficulties are trouble- some, or compliance with tablets is poor.
There are therefore a number of options for the rationalisation of
Mr LN’s medication regimen. A reasonable option would be to convert
each dose of levodopa/carbidopa and entacapone to one tablet of
Stalevo 100/25/200. Ropinirole could also be converted to the XL
preparation, as recommended in the Summary of Product
Characteristics, to one 8 mg tablet daily. These changes would more
than halve the number of tablets taken by Mr LN from 16 per day to
seven.
Which drugs might have contributed to Mr LN’s symptoms?
A20 All the drugs prescribed for Mr LN can cause the psychiatric
complications described.
Levodopa causes a variety of psychiatric symptoms, including
hallucina- tions. Dopamine agonists such as ropinirole can cause
central nervous system effects such as hallucinations and confusion.
These psychiatric complications are especially common in elderly
patients. In addition, progression of the disease itself may contribute
to these symptoms.
A21
What adjustments would you recommend be made to Mr LN’s
medication?
Gradually reduce his dose of ropinirole XL and stop ifnecessary.
The psychiatric complications of most antiparkinsonian drugs are
dose- related and often respond to a reduction in dosage.
It is generally recommended to reduce or eliminate
antiparkinsonian drugs in the following order, corresponding to their
relative propensity to cause psychiatric problems versus degree of
antiparkinsonian activity: anticholinergics, amantadine, MAOBI,
dopamine agonist, levodopa.
Mr LN’s dose of ropinirole should be reduced to the point of
improv- ing his hallucinations without drastically worsening his
parkinsonism, if possible. It should be reduced gradually, as sudden
withdrawal of dopaminergic agents may precipitate a neuroleptic
malignant syndrome. The levodopa dose should only be reduced if
hallucinations persist after elimination of all other antiparkinsonian
agents.
What course of action would you suggest to improve Mr LN’s
A22
symptoms?
Add a low dose of an ‘atypical’ antipsychotic drug, or consider a
cholinesterase inhibitor.
Haloperidol and chlorpromazine are effective antipsychotics but are
not recommended for Parkinson’s patients because of theircapacity
to block striatal dopamine D2 receptors and exacerbate
parkinsonism.
The newer ‘atypical’ antipsychotics (e.g. clozapine, olanzapine,
risperidone, quetiapine) are relatively free of D2-receptor-blocking
potential, and in principle should improve psychotic features without
worsening parkinsonism. The best studied of these is clozapine, which
has been shown to reduce hallucinations without worsening
parkinsonism. However, the potential for clozapine to cause
agranulocytosis (1 – 2% of patients) and the consequent rigorous
monitoring requirements often deter its use. Olanzapine and
risperidone have proved less effective than clozapine in comparative
studies, and have also worsened parkinsonism. The early work with
quetiapine was more hopeful; however, more recent controlled
studies have been negative. When quetiapine is considered, a starting
dose of 12.5 mg at bedtime is recommended and the dose should be
titrated upwards at 3– 5-day intervals until the desired
effect is achieved.
The cholinesterase inhibitors rivastigmine, galantamine and
donepezil, which were developed primarily for Alzheimer’s disease, have
been shown to reduce psychotic features and improve cognition in some
patients with Parkinson’s disease dementias. Most studies have used
rivastigmine and it is the only agent licensed for use in Parkinson’s
disease dementias. A relative cholinergic excess is already present in the
striatum of Parkinson’s disease patients, and thus the use of an agent
likely to support this imbalance further should only be undertaken under
specialist supervision.
What is the long-term outlook for Mr LN?
A23 It is likely that Mr LN’s condition will continue to deteriorate
with time, and that he will experience more ‘off’ time and
increasing dyskinesias. His cognitive function may also decline
further.
Amantadine has been used for the management of dyskinesias based
on its antiglutamate activity; however, its side-effects (confusion,
hallucina- tions) would preclude its use in Mr LN. Other side-effects
of amantadine include ankle swelling and livedo reticularis.
A small number of other therapies can be used at this stage in
patients with Parkinson’s disease, but their risks and financial
implications are considerable.
The use of apomorphine, a potent dopamine agonist licensed for
the treatment of refractory motor fluctuations (‘off’ periods) in IPD,
is a possibility in patients who deteriorate despite maximum
tolerated oral therapy. The drug cannot be given orally and must be
administered subcutaneously. It may be given as a continuous
subcutaneous infusion or as single injections. It causes severe nausea
and vomiting, so 3 days’ pretreatment with domperidone (20 mg
three times daily) is used to minimise this. Domperidone therapy
may be continued until tolerance to this side-effect develops.
Apomorphine is effective within 5– 10 minutes, and patients may
remain in the ‘on’ state for up to 60 minutes. During this time an oral
dose of medication should have taken effect. Many patients are
helped by up to five injections a day, although up to 10 may be
needed in some patients. If the number of injections needed ishigh,
then continuous infusions of apomorphine should be considered. If
the nausea and vomiting can be overcome, apomorphine therapy is
generally well tolerated. Bruising, nodules or abscesses may form at
the site of an infusion, so the site should be changed daily. As Mr LN
has had psychotic features in the past that resolved following
discontinuation of his dopamine agonist, the use of apomorphine
may not be appropriate and may further exacerbate this symptom.
The use of surgical techniques such as subthalamic deep brain
stim- ulation (STN-DBS) may also be precluded in Mr LN by his
previous psychotic symptomatology. The relatively high morbidity
and mortality rate associated with lesioning operations such as
thalamotomy and palli- dotomy has recently made STN-DBS more
favourable. STN-DBS involves the placement of tiny wires into the
subthalamic nucleus (STN). These emit continuous electrical impulses
from a neurostimulator, which is sim- ilar to a heart pacemaker. This
stimulation can have a positive effect on the brain activity involved
in controlling movement, and can improve tremor, stiffness,
slowness and dyskinesia. In addition, with improvement in these
symptoms medication can be reduced, thereby further reducing
dyskinesias. Mr LN may be an appropriate candidate for Duodopa, a gel
formulation of levodopa/carbidopa that is infused directly into the
duodenum. Duodopa is designed to mimic the pharmacokinetic profile
of endogenous dopamine release. The technique, albeit initiated using a
nasogastric tube, requires a percutaneous gastrostomy tube for long-
term administration. Such a procedure in patients with advanced
Parkinson’s disease can be hazardous.
Although there have been many recent advances in the
management of IPD, there is still no cure. Ongoing research is
targeted at neuroprotection strategies: interventions to protect or
rescue vulnerable dopaminergic neurones, and to slow down or stop
disease progression. Gene therapy trials are also currently under
way.

More Related Content

What's hot

parkinsons disease recent updates
parkinsons disease recent updatesparkinsons disease recent updates
parkinsons disease recent updatesNeurologyKota
 
Parkinsons disease V Pharm.D
Parkinsons disease V Pharm.DParkinsons disease V Pharm.D
Parkinsons disease V Pharm.DDr.Sohel Memon
 
Management of early and advanced parkinson disease
Management of early and advanced parkinson diseaseManagement of early and advanced parkinson disease
Management of early and advanced parkinson diseaseNeurologyKota
 
Parkinson's disease an overview
Parkinson's disease an overviewParkinson's disease an overview
Parkinson's disease an overviewMerqurio
 
ANTIPSYCHOTIC TREATMENT.pptx
ANTIPSYCHOTIC TREATMENT.pptxANTIPSYCHOTIC TREATMENT.pptx
ANTIPSYCHOTIC TREATMENT.pptxSmithaRT1
 
Understanding the Brain: Final Project - Parkinson’s Disease
Understanding the Brain: Final Project - Parkinson’s DiseaseUnderstanding the Brain: Final Project - Parkinson’s Disease
Understanding the Brain: Final Project - Parkinson’s DiseaseRachael Shaw
 
Parkinson's PT management
Parkinson's PT managementParkinson's PT management
Parkinson's PT managementRajin Tandan
 
Parkinson's disease
 Parkinson's disease Parkinson's disease
Parkinson's diseaseSunil Pahari
 
Parkinsonism Disease
Parkinsonism DiseaseParkinsonism Disease
Parkinsonism DiseaseVarunsj
 
Parkinson's disease
Parkinson's diseaseParkinson's disease
Parkinson's diseaseraj kumar
 
Management of parkinsons disease
Management of parkinsons diseaseManagement of parkinsons disease
Management of parkinsons diseasesadaf89
 
Parkinsons Disease
Parkinsons DiseaseParkinsons Disease
Parkinsons Diseasetest
 
Advances in schizophrenia
Advances in schizophreniaAdvances in schizophrenia
Advances in schizophreniadrshravan
 
Parkinson's disease
Parkinson's diseaseParkinson's disease
Parkinson's diseaseHIRENGEHLOTH
 

What's hot (20)

parkinsons disease recent updates
parkinsons disease recent updatesparkinsons disease recent updates
parkinsons disease recent updates
 
Parkinsons disease V Pharm.D
Parkinsons disease V Pharm.DParkinsons disease V Pharm.D
Parkinsons disease V Pharm.D
 
Management of early and advanced parkinson disease
Management of early and advanced parkinson diseaseManagement of early and advanced parkinson disease
Management of early and advanced parkinson disease
 
Parkinsons ppt
Parkinsons pptParkinsons ppt
Parkinsons ppt
 
Parkinson's disease an overview
Parkinson's disease an overviewParkinson's disease an overview
Parkinson's disease an overview
 
Parkinson's disease
Parkinson's diseaseParkinson's disease
Parkinson's disease
 
PARKINSON DISEASE
PARKINSON DISEASEPARKINSON DISEASE
PARKINSON DISEASE
 
parkinsons disease
parkinsons diseaseparkinsons disease
parkinsons disease
 
ANTIPSYCHOTIC TREATMENT.pptx
ANTIPSYCHOTIC TREATMENT.pptxANTIPSYCHOTIC TREATMENT.pptx
ANTIPSYCHOTIC TREATMENT.pptx
 
Understanding the Brain: Final Project - Parkinson’s Disease
Understanding the Brain: Final Project - Parkinson’s DiseaseUnderstanding the Brain: Final Project - Parkinson’s Disease
Understanding the Brain: Final Project - Parkinson’s Disease
 
Parkinson's PT management
Parkinson's PT managementParkinson's PT management
Parkinson's PT management
 
Parkinson's diseases
Parkinson's diseasesParkinson's diseases
Parkinson's diseases
 
Parkinson stand 03.12
Parkinson stand 03.12Parkinson stand 03.12
Parkinson stand 03.12
 
Parkinson's disease
 Parkinson's disease Parkinson's disease
Parkinson's disease
 
Parkinsonism Disease
Parkinsonism DiseaseParkinsonism Disease
Parkinsonism Disease
 
Parkinson's disease
Parkinson's diseaseParkinson's disease
Parkinson's disease
 
Management of parkinsons disease
Management of parkinsons diseaseManagement of parkinsons disease
Management of parkinsons disease
 
Parkinsons Disease
Parkinsons DiseaseParkinsons Disease
Parkinsons Disease
 
Advances in schizophrenia
Advances in schizophreniaAdvances in schizophrenia
Advances in schizophrenia
 
Parkinson's disease
Parkinson's diseaseParkinson's disease
Parkinson's disease
 

Similar to Case Series - Parkinson's Disease

Multiple sclerosis case scenario study based
Multiple sclerosis case scenario study basedMultiple sclerosis case scenario study based
Multiple sclerosis case scenario study basedtasbeehalibra
 
case on Pseudo seizures with depression
 case on Pseudo seizures with depression case on Pseudo seizures with depression
case on Pseudo seizures with depressionAnusha Rameshwaram
 
Anti psychotic drugs
Anti psychotic drugsAnti psychotic drugs
Anti psychotic drugsDr Renju Ravi
 
mesial temporal sclerosis and resistant psychosis
mesial temporal sclerosis and resistant psychosismesial temporal sclerosis and resistant psychosis
mesial temporal sclerosis and resistant psychosisShokry Alemam
 
PRN Medications; its justified use: by Dr Prithvi Puwar
PRN Medications; its justified use: by Dr Prithvi PuwarPRN Medications; its justified use: by Dr Prithvi Puwar
PRN Medications; its justified use: by Dr Prithvi PuwarPrithvi Puwar
 
Dq 1 week 5 Nursing homework help.docx
Dq 1 week 5 Nursing homework help.docxDq 1 week 5 Nursing homework help.docx
Dq 1 week 5 Nursing homework help.docxwrite31
 
this dq is due for tomorrow 081818 You are working wit.docx
this dq is due for tomorrow 081818 You are working wit.docxthis dq is due for tomorrow 081818 You are working wit.docx
this dq is due for tomorrow 081818 You are working wit.docxgasciognecaren
 
Nuerodegenrative diseases
Nuerodegenrative diseasesNuerodegenrative diseases
Nuerodegenrative diseasesAnthony21806
 
Anxiety Discussion
Anxiety DiscussionAnxiety Discussion
Anxiety DiscussionJade Abudia
 
Management of adverse effect of antipsychotics 1
Management of adverse effect of antipsychotics 1Management of adverse effect of antipsychotics 1
Management of adverse effect of antipsychotics 1sadaf89
 
REM behaviour disorder and dementia with Lewy bodies
REM behaviour disorder and dementia with Lewy bodiesREM behaviour disorder and dementia with Lewy bodies
REM behaviour disorder and dementia with Lewy bodiesWafik Bahnasy
 
Background This case study presents a 68year old righthan.pdf
Background This case study presents a 68year old righthan.pdfBackground This case study presents a 68year old righthan.pdf
Background This case study presents a 68year old righthan.pdfadvanibagco
 
Quetiapine (Anti psychotic medicine)
Quetiapine (Anti psychotic medicine)Quetiapine (Anti psychotic medicine)
Quetiapine (Anti psychotic medicine)usra_ashraf
 
Parkinson’s disease
Parkinson’s diseaseParkinson’s disease
Parkinson’s diseaseKim Santana
 
Case study on Paranoid Schizophrenia.pptx
Case study on Paranoid Schizophrenia.pptxCase study on Paranoid Schizophrenia.pptx
Case study on Paranoid Schizophrenia.pptxJeeva Anand
 

Similar to Case Series - Parkinson's Disease (20)

parkinson case study.pdf 3.pdf
parkinson case study.pdf 3.pdfparkinson case study.pdf 3.pdf
parkinson case study.pdf 3.pdf
 
Multiple sclerosis case scenario study based
Multiple sclerosis case scenario study basedMultiple sclerosis case scenario study based
Multiple sclerosis case scenario study based
 
case on Pseudo seizures with depression
 case on Pseudo seizures with depression case on Pseudo seizures with depression
case on Pseudo seizures with depression
 
Narcolepsy.docx
Narcolepsy.docxNarcolepsy.docx
Narcolepsy.docx
 
Anti psychotic drugs
Anti psychotic drugsAnti psychotic drugs
Anti psychotic drugs
 
Case Series- Epilepsy
Case Series- EpilepsyCase Series- Epilepsy
Case Series- Epilepsy
 
mesial temporal sclerosis and resistant psychosis
mesial temporal sclerosis and resistant psychosismesial temporal sclerosis and resistant psychosis
mesial temporal sclerosis and resistant psychosis
 
PRN Medications; its justified use: by Dr Prithvi Puwar
PRN Medications; its justified use: by Dr Prithvi PuwarPRN Medications; its justified use: by Dr Prithvi Puwar
PRN Medications; its justified use: by Dr Prithvi Puwar
 
Dq 1 week 5 Nursing homework help.docx
Dq 1 week 5 Nursing homework help.docxDq 1 week 5 Nursing homework help.docx
Dq 1 week 5 Nursing homework help.docx
 
this dq is due for tomorrow 081818 You are working wit.docx
this dq is due for tomorrow 081818 You are working wit.docxthis dq is due for tomorrow 081818 You are working wit.docx
this dq is due for tomorrow 081818 You are working wit.docx
 
Nuerodegenrative diseases
Nuerodegenrative diseasesNuerodegenrative diseases
Nuerodegenrative diseases
 
Anxiety Discussion
Anxiety DiscussionAnxiety Discussion
Anxiety Discussion
 
Management of adverse effect of antipsychotics 1
Management of adverse effect of antipsychotics 1Management of adverse effect of antipsychotics 1
Management of adverse effect of antipsychotics 1
 
REM behaviour disorder and dementia with Lewy bodies
REM behaviour disorder and dementia with Lewy bodiesREM behaviour disorder and dementia with Lewy bodies
REM behaviour disorder and dementia with Lewy bodies
 
Background This case study presents a 68year old righthan.pdf
Background This case study presents a 68year old righthan.pdfBackground This case study presents a 68year old righthan.pdf
Background This case study presents a 68year old righthan.pdf
 
Ms. Tate Case Summary
Ms. Tate Case SummaryMs. Tate Case Summary
Ms. Tate Case Summary
 
Quetiapine (Anti psychotic medicine)
Quetiapine (Anti psychotic medicine)Quetiapine (Anti psychotic medicine)
Quetiapine (Anti psychotic medicine)
 
Parkinson’s disease
Parkinson’s diseaseParkinson’s disease
Parkinson’s disease
 
Case study on Paranoid Schizophrenia.pptx
Case study on Paranoid Schizophrenia.pptxCase study on Paranoid Schizophrenia.pptx
Case study on Paranoid Schizophrenia.pptx
 
Schizophrenia
SchizophreniaSchizophrenia
Schizophrenia
 

More from Shivankan Kakkar

STRUCTURAL ORGANIZATION IN ANIMALS (PART 1)
STRUCTURAL ORGANIZATION IN ANIMALS (PART 1)STRUCTURAL ORGANIZATION IN ANIMALS (PART 1)
STRUCTURAL ORGANIZATION IN ANIMALS (PART 1)Shivankan Kakkar
 
Chemical Coordination and Integration NCERT
Chemical Coordination and Integration NCERTChemical Coordination and Integration NCERT
Chemical Coordination and Integration NCERTShivankan Kakkar
 
Zoology Question Bank for the NEET-UG 2024 Exam
Zoology Question Bank for the NEET-UG 2024 ExamZoology Question Bank for the NEET-UG 2024 Exam
Zoology Question Bank for the NEET-UG 2024 ExamShivankan Kakkar
 
Calculate and Interpret Pharmacokinetic Parameters of a Given Drug
Calculate and Interpret Pharmacokinetic Parameters of a Given DrugCalculate and Interpret Pharmacokinetic Parameters of a Given Drug
Calculate and Interpret Pharmacokinetic Parameters of a Given DrugShivankan Kakkar
 
Effect of Drugs on Dog's Blood Pressure using CAL (Computer aided learning)
Effect of Drugs on Dog's Blood Pressure using CAL (Computer aided learning)Effect of Drugs on Dog's Blood Pressure using CAL (Computer aided learning)
Effect of Drugs on Dog's Blood Pressure using CAL (Computer aided learning)Shivankan Kakkar
 
Dosage Forms and Drug Delivery Systems
Dosage Forms and Drug Delivery SystemsDosage Forms and Drug Delivery Systems
Dosage Forms and Drug Delivery SystemsShivankan Kakkar
 
Essential Medicines List: What You Need to Know
Essential Medicines List: What You Need to KnowEssential Medicines List: What You Need to Know
Essential Medicines List: What You Need to KnowShivankan Kakkar
 
Understanding pA2 and pD2' Values: Calculation and Significance in Pharmacology
Understanding pA2 and pD2' Values: Calculation and Significance in PharmacologyUnderstanding pA2 and pD2' Values: Calculation and Significance in Pharmacology
Understanding pA2 and pD2' Values: Calculation and Significance in PharmacologyShivankan Kakkar
 
Mastering the Art of Setting up an IV Drip: A Step-by-Step Guide
Mastering the Art of Setting up an IV Drip: A Step-by-Step GuideMastering the Art of Setting up an IV Drip: A Step-by-Step Guide
Mastering the Art of Setting up an IV Drip: A Step-by-Step GuideShivankan Kakkar
 
One Liner Pharmacology RECALLS for NEET PG 2022
One Liner Pharmacology RECALLS for NEET PG 2022One Liner Pharmacology RECALLS for NEET PG 2022
One Liner Pharmacology RECALLS for NEET PG 2022Shivankan Kakkar
 
Mastering the Art of Prescribing IV Fluids: A Comprehensive Guide
Mastering the Art of Prescribing IV Fluids: A Comprehensive GuideMastering the Art of Prescribing IV Fluids: A Comprehensive Guide
Mastering the Art of Prescribing IV Fluids: A Comprehensive GuideShivankan Kakkar
 
Alpha Blockers: Mechanisms and Clinical Applications
Alpha Blockers: Mechanisms and Clinical ApplicationsAlpha Blockers: Mechanisms and Clinical Applications
Alpha Blockers: Mechanisms and Clinical ApplicationsShivankan Kakkar
 
Anatomy Question Bank for the NExT / NEET-PG 2023 Exam
Anatomy Question Bank for the NExT / NEET-PG 2023 ExamAnatomy Question Bank for the NExT / NEET-PG 2023 Exam
Anatomy Question Bank for the NExT / NEET-PG 2023 ExamShivankan Kakkar
 
NEET PG Recall- PART I.pptx
NEET PG Recall- PART I.pptxNEET PG Recall- PART I.pptx
NEET PG Recall- PART I.pptxShivankan Kakkar
 
Introduction to pharmacology
Introduction to pharmacologyIntroduction to pharmacology
Introduction to pharmacologyShivankan Kakkar
 
Hepatobiliary pancreatic surgery Set I Review
Hepatobiliary pancreatic surgery Set I ReviewHepatobiliary pancreatic surgery Set I Review
Hepatobiliary pancreatic surgery Set I ReviewShivankan Kakkar
 

More from Shivankan Kakkar (20)

STRUCTURAL ORGANIZATION IN ANIMALS (PART 1)
STRUCTURAL ORGANIZATION IN ANIMALS (PART 1)STRUCTURAL ORGANIZATION IN ANIMALS (PART 1)
STRUCTURAL ORGANIZATION IN ANIMALS (PART 1)
 
Chemical Coordination and Integration NCERT
Chemical Coordination and Integration NCERTChemical Coordination and Integration NCERT
Chemical Coordination and Integration NCERT
 
Zoology Question Bank for the NEET-UG 2024 Exam
Zoology Question Bank for the NEET-UG 2024 ExamZoology Question Bank for the NEET-UG 2024 Exam
Zoology Question Bank for the NEET-UG 2024 Exam
 
Calculate and Interpret Pharmacokinetic Parameters of a Given Drug
Calculate and Interpret Pharmacokinetic Parameters of a Given DrugCalculate and Interpret Pharmacokinetic Parameters of a Given Drug
Calculate and Interpret Pharmacokinetic Parameters of a Given Drug
 
Effect of Drugs on Dog's Blood Pressure using CAL (Computer aided learning)
Effect of Drugs on Dog's Blood Pressure using CAL (Computer aided learning)Effect of Drugs on Dog's Blood Pressure using CAL (Computer aided learning)
Effect of Drugs on Dog's Blood Pressure using CAL (Computer aided learning)
 
Dosage Forms and Drug Delivery Systems
Dosage Forms and Drug Delivery SystemsDosage Forms and Drug Delivery Systems
Dosage Forms and Drug Delivery Systems
 
Antiparkinsonian drugs
Antiparkinsonian drugsAntiparkinsonian drugs
Antiparkinsonian drugs
 
Essential Medicines List: What You Need to Know
Essential Medicines List: What You Need to KnowEssential Medicines List: What You Need to Know
Essential Medicines List: What You Need to Know
 
Understanding pA2 and pD2' Values: Calculation and Significance in Pharmacology
Understanding pA2 and pD2' Values: Calculation and Significance in PharmacologyUnderstanding pA2 and pD2' Values: Calculation and Significance in Pharmacology
Understanding pA2 and pD2' Values: Calculation and Significance in Pharmacology
 
Mastering the Art of Setting up an IV Drip: A Step-by-Step Guide
Mastering the Art of Setting up an IV Drip: A Step-by-Step GuideMastering the Art of Setting up an IV Drip: A Step-by-Step Guide
Mastering the Art of Setting up an IV Drip: A Step-by-Step Guide
 
One Liner Pharmacology RECALLS for NEET PG 2022
One Liner Pharmacology RECALLS for NEET PG 2022One Liner Pharmacology RECALLS for NEET PG 2022
One Liner Pharmacology RECALLS for NEET PG 2022
 
Mastering the Art of Prescribing IV Fluids: A Comprehensive Guide
Mastering the Art of Prescribing IV Fluids: A Comprehensive GuideMastering the Art of Prescribing IV Fluids: A Comprehensive Guide
Mastering the Art of Prescribing IV Fluids: A Comprehensive Guide
 
Alpha Blockers: Mechanisms and Clinical Applications
Alpha Blockers: Mechanisms and Clinical ApplicationsAlpha Blockers: Mechanisms and Clinical Applications
Alpha Blockers: Mechanisms and Clinical Applications
 
CAL Lab Pharmacology
CAL Lab PharmacologyCAL Lab Pharmacology
CAL Lab Pharmacology
 
Anatomy Question Bank for the NExT / NEET-PG 2023 Exam
Anatomy Question Bank for the NExT / NEET-PG 2023 ExamAnatomy Question Bank for the NExT / NEET-PG 2023 Exam
Anatomy Question Bank for the NExT / NEET-PG 2023 Exam
 
NEET PG MAY 2022
NEET PG MAY 2022NEET PG MAY 2022
NEET PG MAY 2022
 
NEET PG Recall- PART I.pptx
NEET PG Recall- PART I.pptxNEET PG Recall- PART I.pptx
NEET PG Recall- PART I.pptx
 
Introduction to pharmacology
Introduction to pharmacologyIntroduction to pharmacology
Introduction to pharmacology
 
Hepatobiliary pancreatic surgery Set I Review
Hepatobiliary pancreatic surgery Set I ReviewHepatobiliary pancreatic surgery Set I Review
Hepatobiliary pancreatic surgery Set I Review
 
Neurology review
Neurology reviewNeurology review
Neurology review
 

Recently uploaded

(Sonam Bajaj) Call Girl in Jaipur- 09257276172 Escorts Service 50% Off with C...
(Sonam Bajaj) Call Girl in Jaipur- 09257276172 Escorts Service 50% Off with C...(Sonam Bajaj) Call Girl in Jaipur- 09257276172 Escorts Service 50% Off with C...
(Sonam Bajaj) Call Girl in Jaipur- 09257276172 Escorts Service 50% Off with C...indiancallgirl4rent
 
VIP Kolkata Call Girl New Town 👉 8250192130 Available With Room
VIP Kolkata Call Girl New Town 👉 8250192130  Available With RoomVIP Kolkata Call Girl New Town 👉 8250192130  Available With Room
VIP Kolkata Call Girl New Town 👉 8250192130 Available With Roomdivyansh0kumar0
 
Bangalore call girl 👯‍♀️@ Simran Independent Call Girls in Bangalore GIUXUZ...
Bangalore call girl  👯‍♀️@ Simran Independent Call Girls in Bangalore  GIUXUZ...Bangalore call girl  👯‍♀️@ Simran Independent Call Girls in Bangalore  GIUXUZ...
Bangalore call girl 👯‍♀️@ Simran Independent Call Girls in Bangalore GIUXUZ...Gfnyt
 
❤️♀️@ Jaipur Call Girls ❤️♀️@ Meghna Jaipur Call Girls Number CRTHNR Call G...
❤️♀️@ Jaipur Call Girls ❤️♀️@ Meghna Jaipur Call Girls Number CRTHNR   Call G...❤️♀️@ Jaipur Call Girls ❤️♀️@ Meghna Jaipur Call Girls Number CRTHNR   Call G...
❤️♀️@ Jaipur Call Girls ❤️♀️@ Meghna Jaipur Call Girls Number CRTHNR Call G...Gfnyt.com
 
Basics of Anatomy- Language of Anatomy.pptx
Basics of Anatomy- Language of Anatomy.pptxBasics of Anatomy- Language of Anatomy.pptx
Basics of Anatomy- Language of Anatomy.pptxAyush Gupta
 
Vip sexy Call Girls Service In Sector 137,9999965857 Young Female Escorts Ser...
Vip sexy Call Girls Service In Sector 137,9999965857 Young Female Escorts Ser...Vip sexy Call Girls Service In Sector 137,9999965857 Young Female Escorts Ser...
Vip sexy Call Girls Service In Sector 137,9999965857 Young Female Escorts Ser...Call Girls Noida
 
Call Girls Thane Just Call 9907093804 Top Class Call Girl Service Available
Call Girls Thane Just Call 9907093804 Top Class Call Girl Service AvailableCall Girls Thane Just Call 9907093804 Top Class Call Girl Service Available
Call Girls Thane Just Call 9907093804 Top Class Call Girl Service AvailableDipal Arora
 
Call Girls Hyderabad Just Call 9907093804 Top Class Call Girl Service Available
Call Girls Hyderabad Just Call 9907093804 Top Class Call Girl Service AvailableCall Girls Hyderabad Just Call 9907093804 Top Class Call Girl Service Available
Call Girls Hyderabad Just Call 9907093804 Top Class Call Girl Service AvailableDipal Arora
 
Local Housewife and effective ☎️ 8250192130 🍉🍓 Sexy Girls VIP Call Girls Chan...
Local Housewife and effective ☎️ 8250192130 🍉🍓 Sexy Girls VIP Call Girls Chan...Local Housewife and effective ☎️ 8250192130 🍉🍓 Sexy Girls VIP Call Girls Chan...
Local Housewife and effective ☎️ 8250192130 🍉🍓 Sexy Girls VIP Call Girls Chan...Russian Call Girls Amritsar
 
VIP Call Girls Sector 67 Gurgaon Just Call Me 9711199012
VIP Call Girls Sector 67 Gurgaon Just Call Me 9711199012VIP Call Girls Sector 67 Gurgaon Just Call Me 9711199012
VIP Call Girls Sector 67 Gurgaon Just Call Me 9711199012Call Girls Service Gurgaon
 
Dehradun Call Girls Service 08854095900 Real Russian Girls Looking Models
Dehradun Call Girls Service 08854095900 Real Russian Girls Looking ModelsDehradun Call Girls Service 08854095900 Real Russian Girls Looking Models
Dehradun Call Girls Service 08854095900 Real Russian Girls Looking Modelsindiancallgirl4rent
 
indian Call Girl Panchkula ❤️🍑 9907093804 Low Rate Call Girls Ludhiana Tulsi
indian Call Girl Panchkula ❤️🍑 9907093804 Low Rate Call Girls Ludhiana Tulsiindian Call Girl Panchkula ❤️🍑 9907093804 Low Rate Call Girls Ludhiana Tulsi
indian Call Girl Panchkula ❤️🍑 9907093804 Low Rate Call Girls Ludhiana TulsiHigh Profile Call Girls Chandigarh Aarushi
 
Call Girl In Zirakpur ❤️♀️@ 9988299661 Zirakpur Call Girls Near Me ❤️♀️@ Sexy...
Call Girl In Zirakpur ❤️♀️@ 9988299661 Zirakpur Call Girls Near Me ❤️♀️@ Sexy...Call Girl In Zirakpur ❤️♀️@ 9988299661 Zirakpur Call Girls Near Me ❤️♀️@ Sexy...
Call Girl In Zirakpur ❤️♀️@ 9988299661 Zirakpur Call Girls Near Me ❤️♀️@ Sexy...Sheetaleventcompany
 
Call Girls Service Charbagh { Lucknow Call Girls Service 9548273370 } Book me...
Call Girls Service Charbagh { Lucknow Call Girls Service 9548273370 } Book me...Call Girls Service Charbagh { Lucknow Call Girls Service 9548273370 } Book me...
Call Girls Service Charbagh { Lucknow Call Girls Service 9548273370 } Book me...gragteena
 
Russian Escorts Aishbagh Road * 9548273370 Naughty Call Girls Service in Lucknow
Russian Escorts Aishbagh Road * 9548273370 Naughty Call Girls Service in LucknowRussian Escorts Aishbagh Road * 9548273370 Naughty Call Girls Service in Lucknow
Russian Escorts Aishbagh Road * 9548273370 Naughty Call Girls Service in Lucknowgragteena
 
Russian Call Girls Kota * 8250192130 Service starts from just ₹9999 ✅
Russian Call Girls Kota * 8250192130 Service starts from just ₹9999 ✅Russian Call Girls Kota * 8250192130 Service starts from just ₹9999 ✅
Russian Call Girls Kota * 8250192130 Service starts from just ₹9999 ✅gragmanisha42
 
Krishnagiri call girls Tamil aunty 7877702510
Krishnagiri call girls Tamil aunty 7877702510Krishnagiri call girls Tamil aunty 7877702510
Krishnagiri call girls Tamil aunty 7877702510Vipesco
 
💚😋Kolkata Escort Service Call Girls, ₹5000 To 25K With AC💚😋
💚😋Kolkata Escort Service Call Girls, ₹5000 To 25K With AC💚😋💚😋Kolkata Escort Service Call Girls, ₹5000 To 25K With AC💚😋
💚😋Kolkata Escort Service Call Girls, ₹5000 To 25K With AC💚😋Sheetaleventcompany
 

Recently uploaded (20)

(Sonam Bajaj) Call Girl in Jaipur- 09257276172 Escorts Service 50% Off with C...
(Sonam Bajaj) Call Girl in Jaipur- 09257276172 Escorts Service 50% Off with C...(Sonam Bajaj) Call Girl in Jaipur- 09257276172 Escorts Service 50% Off with C...
(Sonam Bajaj) Call Girl in Jaipur- 09257276172 Escorts Service 50% Off with C...
 
VIP Kolkata Call Girl New Town 👉 8250192130 Available With Room
VIP Kolkata Call Girl New Town 👉 8250192130  Available With RoomVIP Kolkata Call Girl New Town 👉 8250192130  Available With Room
VIP Kolkata Call Girl New Town 👉 8250192130 Available With Room
 
Bangalore call girl 👯‍♀️@ Simran Independent Call Girls in Bangalore GIUXUZ...
Bangalore call girl  👯‍♀️@ Simran Independent Call Girls in Bangalore  GIUXUZ...Bangalore call girl  👯‍♀️@ Simran Independent Call Girls in Bangalore  GIUXUZ...
Bangalore call girl 👯‍♀️@ Simran Independent Call Girls in Bangalore GIUXUZ...
 
❤️♀️@ Jaipur Call Girls ❤️♀️@ Meghna Jaipur Call Girls Number CRTHNR Call G...
❤️♀️@ Jaipur Call Girls ❤️♀️@ Meghna Jaipur Call Girls Number CRTHNR   Call G...❤️♀️@ Jaipur Call Girls ❤️♀️@ Meghna Jaipur Call Girls Number CRTHNR   Call G...
❤️♀️@ Jaipur Call Girls ❤️♀️@ Meghna Jaipur Call Girls Number CRTHNR Call G...
 
#9711199012# African Student Escorts in Delhi 😘 Call Girls Delhi
#9711199012# African Student Escorts in Delhi 😘 Call Girls Delhi#9711199012# African Student Escorts in Delhi 😘 Call Girls Delhi
#9711199012# African Student Escorts in Delhi 😘 Call Girls Delhi
 
Model Call Girl in Subhash Nagar Delhi reach out to us at 🔝9953056974🔝
Model Call Girl in Subhash Nagar Delhi reach out to us at 🔝9953056974🔝Model Call Girl in Subhash Nagar Delhi reach out to us at 🔝9953056974🔝
Model Call Girl in Subhash Nagar Delhi reach out to us at 🔝9953056974🔝
 
Basics of Anatomy- Language of Anatomy.pptx
Basics of Anatomy- Language of Anatomy.pptxBasics of Anatomy- Language of Anatomy.pptx
Basics of Anatomy- Language of Anatomy.pptx
 
Vip sexy Call Girls Service In Sector 137,9999965857 Young Female Escorts Ser...
Vip sexy Call Girls Service In Sector 137,9999965857 Young Female Escorts Ser...Vip sexy Call Girls Service In Sector 137,9999965857 Young Female Escorts Ser...
Vip sexy Call Girls Service In Sector 137,9999965857 Young Female Escorts Ser...
 
Call Girls Thane Just Call 9907093804 Top Class Call Girl Service Available
Call Girls Thane Just Call 9907093804 Top Class Call Girl Service AvailableCall Girls Thane Just Call 9907093804 Top Class Call Girl Service Available
Call Girls Thane Just Call 9907093804 Top Class Call Girl Service Available
 
Call Girls Hyderabad Just Call 9907093804 Top Class Call Girl Service Available
Call Girls Hyderabad Just Call 9907093804 Top Class Call Girl Service AvailableCall Girls Hyderabad Just Call 9907093804 Top Class Call Girl Service Available
Call Girls Hyderabad Just Call 9907093804 Top Class Call Girl Service Available
 
Local Housewife and effective ☎️ 8250192130 🍉🍓 Sexy Girls VIP Call Girls Chan...
Local Housewife and effective ☎️ 8250192130 🍉🍓 Sexy Girls VIP Call Girls Chan...Local Housewife and effective ☎️ 8250192130 🍉🍓 Sexy Girls VIP Call Girls Chan...
Local Housewife and effective ☎️ 8250192130 🍉🍓 Sexy Girls VIP Call Girls Chan...
 
VIP Call Girls Sector 67 Gurgaon Just Call Me 9711199012
VIP Call Girls Sector 67 Gurgaon Just Call Me 9711199012VIP Call Girls Sector 67 Gurgaon Just Call Me 9711199012
VIP Call Girls Sector 67 Gurgaon Just Call Me 9711199012
 
Dehradun Call Girls Service 08854095900 Real Russian Girls Looking Models
Dehradun Call Girls Service 08854095900 Real Russian Girls Looking ModelsDehradun Call Girls Service 08854095900 Real Russian Girls Looking Models
Dehradun Call Girls Service 08854095900 Real Russian Girls Looking Models
 
indian Call Girl Panchkula ❤️🍑 9907093804 Low Rate Call Girls Ludhiana Tulsi
indian Call Girl Panchkula ❤️🍑 9907093804 Low Rate Call Girls Ludhiana Tulsiindian Call Girl Panchkula ❤️🍑 9907093804 Low Rate Call Girls Ludhiana Tulsi
indian Call Girl Panchkula ❤️🍑 9907093804 Low Rate Call Girls Ludhiana Tulsi
 
Call Girl In Zirakpur ❤️♀️@ 9988299661 Zirakpur Call Girls Near Me ❤️♀️@ Sexy...
Call Girl In Zirakpur ❤️♀️@ 9988299661 Zirakpur Call Girls Near Me ❤️♀️@ Sexy...Call Girl In Zirakpur ❤️♀️@ 9988299661 Zirakpur Call Girls Near Me ❤️♀️@ Sexy...
Call Girl In Zirakpur ❤️♀️@ 9988299661 Zirakpur Call Girls Near Me ❤️♀️@ Sexy...
 
Call Girls Service Charbagh { Lucknow Call Girls Service 9548273370 } Book me...
Call Girls Service Charbagh { Lucknow Call Girls Service 9548273370 } Book me...Call Girls Service Charbagh { Lucknow Call Girls Service 9548273370 } Book me...
Call Girls Service Charbagh { Lucknow Call Girls Service 9548273370 } Book me...
 
Russian Escorts Aishbagh Road * 9548273370 Naughty Call Girls Service in Lucknow
Russian Escorts Aishbagh Road * 9548273370 Naughty Call Girls Service in LucknowRussian Escorts Aishbagh Road * 9548273370 Naughty Call Girls Service in Lucknow
Russian Escorts Aishbagh Road * 9548273370 Naughty Call Girls Service in Lucknow
 
Russian Call Girls Kota * 8250192130 Service starts from just ₹9999 ✅
Russian Call Girls Kota * 8250192130 Service starts from just ₹9999 ✅Russian Call Girls Kota * 8250192130 Service starts from just ₹9999 ✅
Russian Call Girls Kota * 8250192130 Service starts from just ₹9999 ✅
 
Krishnagiri call girls Tamil aunty 7877702510
Krishnagiri call girls Tamil aunty 7877702510Krishnagiri call girls Tamil aunty 7877702510
Krishnagiri call girls Tamil aunty 7877702510
 
💚😋Kolkata Escort Service Call Girls, ₹5000 To 25K With AC💚😋
💚😋Kolkata Escort Service Call Girls, ₹5000 To 25K With AC💚😋💚😋Kolkata Escort Service Call Girls, ₹5000 To 25K With AC💚😋
💚😋Kolkata Escort Service Call Girls, ₹5000 To 25K With AC💚😋
 

Case Series - Parkinson's Disease

  • 1. Parkinson’s disease Discussion by ShivankanKakkar, MD Long Case Day 1 Mr LN, a 67-year-old retired IAS Officer, was admitted to the neurology ward. His presenting complaints included difficulty in getting up from chairs and initiating walking. Eighteen months earlier he had noticed that his self-winding watch, which he wore on his
  • 2. right wrist, was consistently losing time. When moved to his left wrist, the watch kept perfect time. Friends had pointed out that he had developed a limp, dragging his right foot. These symptoms had worsened over the previous 18 months, and he had also developed a resting tremor in his right hand and leg. He had noticed changes in his bowel habits many years previously. His general practitioner (GP) suspected Parkinson’s disease and referred Mr LN to a neurologist for confirmation of the diagnosis. Q1 What are the main symptoms of Parkinson’s disease? Q2 Is it possible to make a definitive diagnosis of idiopathic Parkinson’s
  • 3. disease (IPD)? Q3 What biochemical defects are thought to be present in a patient such as Mr LN? Q4 Outline a treatment care plan for Mr LN. Q5 Which drugs are usually considered for the initial treatment of patients with IPD and what are their modes of action? Day 2 Mr LN was started onropinirole. Q6 Do you agree with this choice? Q7 How should Mr LN’s ropinirole therapy be adjusted to optimise his response? Q8 What counselling points would you highlight to a patient commencing ropinirole?
  • 4. Day 13 Mr LN was discharged on ropinirole 500 micrograms three times daily and instructed to titrate the dose as directed. Mr LN’s GP was sent a letter describing his recent admission and advice on how to titrate the dose of ropinirole further. Month 18 Mr LN was admitted for reassessment. Despite never completely abating, Mr LN’s symptoms of bradykinesia and rigidity had temporarily improved following initiation and up-titration of ropinirole; however, recently Mr LN had experienced a worsening of these symptoms, and his wife reported that he had become rather obsessive about cleaning out the garden shed, performing this task
  • 5. on an almost daily basis. On admission he was taking ropinirole 6 mg three times daily. Q9 What recommendations would you make with regard to Mr LN’s drug therapy? It was decided to start Mr LN on Sinemet Plus 125 mg three times daily and simultaneously to reduce the dose of his dopamine agonist. Q10 How can the adverse effects of levodopa be minimised? Month 18, Day 8 Mr LN was discharged. He was now stabilised on
  • 6. Sinemet Plus, two tablets three times daily, and ropinirole 4 mg three times daily and had experienced a significant improvement in his motor function. Month 66 Mr LN was again admitted for reassessment. His therapy and symptoms had remained stable on the Sinemet and ropinirole until recently, when he had started to notice his arms shaking about 1 hour after each dose of Sinemet Plus. This lasted for about 45 minutes before stopping. Mr LN also complained of painful dystonic cramps at night, which caused him to wake early most mornings. He had attempted to counter this by taking an extra Sinemet Plus tablet shortly before going to bed, which sometimes helped. His wife added
  • 7. that he was becoming profoundly immobile up to 2 hours before his Sinemet doses were due during the day. On examination he was found to have a mask-like face and a resting tremor in both hands and legs. ‘Cogwheel’ rigidity was found in all four limbs. He had difficulty initiating speech and his voice was very quiet. He struggled to rise from his chair, and had a characteristic ‘parkinsonian shuffle’ when walking. When his shoulders were pulled forward from standing he staggered forward and had to be supported to stop him falling. Mr LN complained of occasional falls and was not confident to leave the house. He relied on his wife to do everything for him. He
  • 8. admitted to being depressed about his condition. There was no other medical history of note. All routine laboratory tests were within normal ranges. His drug therapy was ropinirole 4 mg three times daily plus two Sinemet Plus tablets at 8 a.m., 2 p.m. and 7 p.m. and, occasionally, one Sinemet Plus tablet at11 p.m. Q11 Which of the long-term complications of levodopa therapy is Mr LN suffering from? Q12 What alterations to Mr LN’s drug therapy would you recommend in order to try to minimise these effects? Q13 How could you monitor and assess Mr LN’s response to these changes? Q14 Can you suggest any non-drug management that might benefit Mr LN? Mr LN’s drug therapy was changed to one Madopar dispersible
  • 9. 62.5 mgtablet on waking around 7 a.m., and one Sinemet Plus tablet at 10 a.m., 1 p.m., 4 p.m. and 7 p.m. In addition, he was prescribed one Sinemet CR tablet at 10 p.m. The nursing staff were asked to complete an hourly ‘on– off’ chart. Month 66, Day 6 Mr LN’s mobility and dyskinesias were improved; however, he remained depressed about his condition. Q15 Would Mr LN benefit from an antidepressant? Q16 If so, which would you choose?
  • 10. Completed on/off chart Time Day 1 Day 5 7 a.m. off off (dose) 8 a.m. off (dose) on (dyskinesia) 9 a.m. on on (dyskinesia) 10 a.m. on off (dose) (dyskinesia) 11 a.m. on on (dyskinesia) 12 noon off on 1 p.m. off on (dose) 2 p.m. off (dose) on (dyskinesia) 3 p.m. on on (dyskinesia)
  • 11. Month 66, Day 8 Mr LN had noticed a significant reduction in the shaking of his arms following his recent regimen change; however, he was still becoming considerably immobile about 1 hour before his dose of Sinemet. His sleep had improved so that he was getting at least 6 continuous hours’ sleep and no longer woke in pain from cramps. The consultant neurologist recommended commencing entacapone. Q17 Why has the consultant neurologist recommended entacapone, and does this necessitate the adjustment of Mr LN’s other IPD therapy? Q18 What counselling points would you highlight to a patient commencing entacapone?
  • 12. Month 66, Day 11 During the ward round, you noted that Mr LN’s morning medication was still on his bedside table. His medication now comprised one Madopar dispersible 62.5 mg tablet on waking around 7 a.m., one Sinemet Plus tablet at 10 a.m., 1 p.m., 4 p.m. and 7 p.m. and a Sinemet CR tablet at 10 p.m. In addition, he was taking one entacapone tablet alongside each dose of Sinemet Plus and 3 mg ropinirole three times a day (made up of a 1 mg and a 2 mg tablet for each dose). His regimen thus comprised a total of 16 tablets per day. On questioning, Mr LN informed you that, although he was feeling better, he was concerned at the number of tablets he was taking and
  • 13. feared that he would have problems remembering to take them all when he returned home. He also found the new tablet quite hard to swallow because of its size. Q19 In view of Mr LN’s concerns, what options are there for rationalising his medications? Month 75 Mr LN was admitted as an emergency by his GP, having developed visual and auditory hallucinations over the previous week. He was hearing voices talking about him, threatening to kill him. He was also seeing insects crawling up the walls and burrowing into his skin. On examination he was clearly distressed and very frightened. His medication on admission was one Madopar dispersible tablet on
  • 14. waking, one Stalevo 100/25/200 tablet four times daily, one Sinemet CR tablet at night and oneropinirole XL tablet 8 mg daily. Q20 Which drugs might have contributed to Mr LN’s symptoms? Q21 What adjustments would you recommend be made to Mr LN’s medication? Month 75, Day 7 The recommendations had been carried out. Mr LN’s visual hallucinations had improved, but he was still experiencing
  • 15. distressing auditory hallucinations and the control of his symptoms had deteriorated, such that he was experiencing frequent ‘off’ periods. Q22 What course of action would you suggest to improve Mr LN’s symptoms? Mr LN was started on rivastigmine 1.5 mg twice daily. The dose was increased over the next 10 days to 4.5 mg twice daily. His dopaminergic therapy was adjusted to one Stalevo 100/25/200 tablet five times daily, one Sinemet CR tablet at night and one Madopar 62.5 mg dispersible tablet upon waking. His ropinirole XL therapy was discontinued. His hallucinations resolved and acceptable control of his Parkinson’s symptoms was achieved. He was discharged on this regimen. Q23 What is the long-term outlook for Mr LN?
  • 16. The main symptoms in patients with Parkinson’s disease are tremor, rigidity, bradykinesia (slowness of movement), akinesia (loss of movement) and postural abnormalities. What are the main symptoms of Parkinson’s disease? A1 The onset of Parkinson’s disease is usually insidious and progression slow. Many patients first notice a resting tremor. This usuallyinitially affects the hands and may be unilateral. The tremor disappears on movement and during sleep, and may be worse under stress. The patient is usually over 50 years old on presentation. Rigidity manifests as an increased resistance to passive movement
  • 17. and is classically termed ‘cogwheel’ rigidity, with a ratchet-like phe- nomenon felt at the wrist on passive movement of the hand. Bradykinesia manifests as a general slowness in movement. Together with the rigidity, it is responsible for the typical abnormalities of gait: difficulty in starting and finishing steps, resulting in shuffling; a stooped head; flexed neck, upper extremities and knees; and a lack of normal arm swing. A wide range of non- motor symptoms (NMS) have also been described in Parkinson’s disease, all of which can have a significant impact on quality of life. These include bowel and bladder problems, fatigue, pain, sleep disorders and autonomic dysfunction, in addition to difficulty in swallowing and speech alteration, drooling of saliva and olfactory
  • 18. disturbance. Loss of postural reflexes leads to postural imbalance and sometimes to frequent falls. NMS are common and can occur at all stages of Parkinson’s disease, including long beforediagnosis. Symptoms increase in number and severity as the disease progresses. Until recently, there has been a tendency for healthcare professionals to overlook these features in favour of the more apparent motor symptoms. Is it possible to make a definitive diagnosis of idiopathic Parkinson’s disease (IPD)? A2 At present the only way of making a definite diagnosis of IPD is by postmortem study of the brain.
  • 19. IPD is the most common cause of parkinsonism, accounting for approxiimately 75% of cases presenting to neurologists. Other causes include other neurodegenerative diseases such as progressive supranuclear palsy (PSP) and multiple system atrophy (MSA), intoxication with heavy metals, treatment with therapeutic drugs (neuroleptics, metoclopramide) and chronic cerebrovascular disease. The definitive diagnosis of Parkinson’s disease is based on characteristic neuropathological findings of Lewy bodies and neuronal loss in the substantia nigra and other brainstem nuclei. Studies have shown that only 65– 75% of patients diagnosed as having early Parkinson’s disease had the characteristic findings at postmortem.
  • 20. Current opinion and guidelines recommend that all patients with a ‘suspected’ diagnosis of IPD must be referred untreated to a specialist who can reliably differentiate between IPD and other parkinsonian syndromes. Previously the response to a ‘challenge’ of levodopa or dopaminergic agents has been used for diagnostic purposes; however the National Institute for Health and Clinical Excellence (NICE) guidance for Parkinson’s disease does not advocate that this be performed routinely, although guidelines vary worldwide. Acute challenge testing is similar to, but not better than, chronic levodopa therapy in terms of diagnostic accuracy, but is associated with adverse effects and extra costs.
  • 21. The use of imaging techniques is increasing in this field. Single photon emission computed tomography (SPECT) with DatSCAN, a radi- olabelled cocaine derivative, can be used to measure the amount of dopamine-releasing neurones in the brain. This type of imaging can aid differentiation between parkinsonian and non- parkinsonian syndromes, but it is unable to distinguish between IPD, MSA and PSP.
  • 22. What biochemical defects are thought to be present in a patient A3 such as Mr LN? Several biochemical defects are thought to be present in patients with Parkinson’s disease. A combination of cholinergic (excitatory) and dopaminergic (inhibitory) mechanisms acting in the striatal tracts of the basal ganglia of the brain are thought to be responsible for the smooth control of voluntary movements. Imbalances in the neurotransmitters lead to movement disorders. In patients with Parkinson’s disease, dopamine concentrations in the three major parts of the basal ganglia are reduced to a fraction of
  • 23. normal. Compensatory mechanisms operate and symptoms are not noted until a severe loss (80%) of dopaminergic neurones has occurred. The severity of some symptoms, such as bradykinesia, has been found to correlate with striatal dopamine levels; however, abnormalities of other neurotrans- mitters, including norepinephrine (noradrenaline), 5-hydroxytryptamine (serotonin) and gamma- aminobutyric acid, have also been reported. The full relevance of these changes is unclear.
  • 24. Outline a treatment care plan for Mr LN. A4 Thegoals of symptomatic drug treatment are to help the patient function independently for as long as possible, and to achieve this with the minimum of adverse effects. Patients with Parkinson’s disease have a chronic deteriorating condition which will result in lifelong drug therapy of increasing complexity. A long-term treatment care plan would include: (a) Involving Mr LN in the choice of appropriate initial therapy. (b) Ensuring the development of appropriate treatment outcome mea- sures and a suitable treatment-monitoring programme. (c) Ensuring that the patient understands the role of drugs inthe
  • 25. symptomatic treatment of the disease and their possible adverse effects. (d) Ensuring that the patient and carers understand the importance of adherence and timing of drug doses. (e) Anticipating problems such as the potential for nausea and vomiting with levodopa preparations, and offering appropriate advice on their prevention. (f) Counselling the patient and carers about drugs that should be avoided in Parkinson’s disease. These include medicines which act as dopamine antagonists, such as metoclopramide and the older antipsychotics (e.g. chlorpromazine, haloperidol). (g) As the disease progresses and more drugs are added to the
  • 26. regimen, medicine taking may become problematic and advice on methods for improving and maintaining adequate adherence should be given. (h) As most patients with IPD will be elderly, the general considerations given to elderly patients should also be applied. As treatment may continue for many years and become increasingly complex, continuity of pharmaceutical care is an issue and consideration should be given to a personal, individualised patient record that can be used by pharmacists at various stages of the disease. The Non-pharmacological considerations include education on the disease itself, advice on diet, exercise programmes, and other areas such as driving, alcohol intake and recreational activities.
  • 27. Which drugs are usually considered for the initial treatment of patients with IPD and what are their modes of action? A5 Initial drug therapy as recommended by NICE guidelines forboth early- and late-onset IPD will usually be chosen from the following: levodopa preparations; a dopamine agonist; or a monoamine oxidase B inhibitor (MAOBI). There is still considerable debate about the best initial choice of therapy for patients with IPD, and also when to start symptomatic treatment. Most experts now advocate early treatment to provide patients with maximal clinical benefit at the start of their illness. Most Parkinson’s disease specialists start treatment when a patient’s
  • 28. symptoms begin to interfere with their lifestyle. What constitutes this will vary from patient to patient, but may include impairment of activities of daily living (ADL), threatened loss of employment or gait disturbance with a risk of falling. When a decision is made to initiate treatment, the age of the patient, the degree and type of symptoms and the patient’s expectations will influence drug choice. Patients should be made aware that drug treatment can provide symptomatic relief, but that there is at present no way of halting the progression of the disease. Levodopa is the most effective drug in the symptomatic management of Parkinson’s disease, and virtually all patients will experience meaningful benefit; however, it can cause significant short- and long-
  • 29. term adverse effects. The rationale for early use of dopamine agonists is that they provide some benefit in early disease, but are significantly less likely to lead to the development of motor complications, particularly dyskinesias. Recent studies comparing levodopa with pramipexole in early disease have shown that levodopa more effectively manages motor symptoms, but is related to a greater incidence of motor complications. Treatment withdrawal through adverse effects (e.g. somnolence and peripheral oedema) was higher in the pramipexole group. The incidence of impulse control disorders with dopamine agonists is reported to be around 17% and can have significant impact. The once daily formulations of pramipexole and ropinirole
  • 30. have an advantage not only in compliance, but also in managing symptoms for young patients who are still working, and who donot wish for the higher frequency of dosing required with levodopa. While it is an individual decision between the patient and the consultant, initiation of a dopamine agonist in patients who require this but who still have relatively mild symptoms may be appropriate. Patients with more severe symptoms and those over 70 (in whom the development of motor complications is less likely) should probably be started on a levodopa preparation. Anticholinergic drugs are now rarely used for the treatment of Parkinson’s disease. They provide some relief of tremor but are of little value in the treatment of other features, such as rigidity and bradykinesia. In addition, adverse effects are
  • 31. common. These include peripheral effects such as dry mouth, blurred vision and constipation, as well as potentially serious central effects, including confusion and hallucinations. Their use is best reserved for younger patients (under 60), in whom resting tremor is the predominant feature. Anticholinergic drugs are thought to act by correcting the relative central cholinergic excess brought about by dopamine deficiency. They are not recommended as first-line treatment. The MAOBIs selegiline and rasagiline selectively inhibit monoamine oxidase B, one of the enzyme systems which break down dopamine. The action of endogenous dopamine is thus augmented. Selegiline has a mild antiparkinsonian effect and is sometimes used either alone in early disease, or later to potentiate the action of
  • 32. levodopa preparations. Selegi- line is metabolised to amfetamine derivatives, so it may have an alerting effect, especially at night, when it can cause insomnia, vivid dreams andnightmares. Rasagiline, although very similar in chemical structure, isnot metabolised to amfetamine derivatives and so is potentially free of alerting side-effects. MAOBIs have been investigated for possession of neuroprotective properties, but this has not yet been proven. Dopamine deficiency cannot be rectified by the administration of dopamine, because dopamine does not cross the blood– brain barrier. Lev- odopa does cross the blood– brain barrier and is converted to dopamine in the basal ganglia. Levodopa is thus thought to act primarily
  • 33. by increasing brain dopamine concentrations. If levodopa is administered alone, over 95% of a dose is decarboxylated to dopamine peripherally, which results in reduced amounts being available to cross the blood– brain barrier and problematic peripheral side-effects such as nausea, vomiting and postural hypotension. Levodopa is therefore now almost always administered with a dopa-decarboxylase inhibitor, either carbidopa or benserazide. These dopa-decarboxylase inhibitors do not cross the blood– brain barrier and so smaller daily doses of levodopa can be administered, thereby reducing the incidence of peripheral side-effects. Levodopa therapy is particularly helpful in controlling symptoms of bradykinesia or akinesia.
  • 34. Do you agree with this choice? A6 Yes. The choice of initial therapy is based on many different factors that will vary from patient to patient. The useof a dopamine agonist is indeed appropriate in this situation, as would be the use of levodopa. The dopamine agonists can be split into two different categories: ergot and non-ergot. Ergot-derived dopamine agonists include the older- generation products bromocriptine, lisuride, pergolide and cabergoline. Non-ergot- derived agonists include the newer agents pramipexole, ropinirole and rotigotine.
  • 35. Use of the ergot-derived agonists has declined considerably over the last few years as a result of increased concerns about long-term side- effects, specifically cardiac fibrosis. There are clear monitoring guidelines for patients who remain on these agents. Critical appraisal of the literature evaluating the two non-ergot- derived oral agonists provides no evidence to prefer one over the other. Rotigotine, the transdermal patch formulation, although considered less potent than the other non-ergot-derived dopamine agonists, provides con- tinuous dopaminergic stimulation (CDS) over a 24-hour period. Although it is suggested that 24-hour continuous drug release more closely resembles endogenous dopamine release and may reduce the risk of motor fluctu- ations and dyskinesias developing, there are at
  • 36. present no firm clinical data to support this. Ropinirole XL and pramipexole PR are also available as once-daily preparations. The initial requirement to stabilise patients with immediate-release ropinirole prior to starting once-daily (XL) has now been removed, and where local formulary allows, prolonged-release formulations can be used from initiation. How should Mr LN’s ropinirole therapy be adjusted to optimise his response? A7 Like any dopaminergic therapy, ropinirole must be started ata low dose and increased gradually in order to reduce the incidence of side-effects and to establish the lowest effective dose.
  • 37. In order to facilitate gradual up-titration, ropinirole is available as a ‘starter pack’ that contains all the medication required for a stepwise dose titration over a 4-week period. A ‘follow-on’ pack allows further stepwise titration over a similar period to a therapeutic dose. This dose can then be further increased as required and tolerated by the patient. Rotigotine patches are also available in a starter pack format. Pramipexole doses are often referred to in both the salt and the base form. It is important to be aware of this when referring to doses with patients and other healthcare professionals; there are a number of information materials available to aid patients and reduce confusion.
  • 38. What counselling points would you highlight to apatient A8 commencing ropinirole? Patients should be warned about the potential for dopamine agonists to cause impulse control disorders. In addition they should be counselled on the risk of excessive daytime somnolence and the implications of this on driving/operating machinery. What recommendations would you make with regard to Mr LN’s A9 drug therapy? Mr LN should at this stage be commenced on a low dose of levodopa as an adjunct to the dopamine agonist. The dopamine agonist dose should be reduced in view of his history of pund-
  • 39. ing (the obsessive and repetitive perfomance of a useless task). The levodopa dose should be gradually increased, with decisions based on symptom relief and adverse effects. Levodopa and dopa-decarboxylase inhibitors are marketed in the UK as Sinemet (levodopa plus carbidopa: co-careldopa) and Madopar (levodopa plus benserazide: co-beneldopa). There are six Madopar preparations and six Sinemet preparations available in a variety of levodopa strengths and formulations (standard, dispersible and controlled release). The potential for confusion among clinicians and pharmacists is considerable, and care must be taken to ensure that the patient receives the intended preparation. At the start of levodopa
  • 40. therapy the effects of a dose usually last for 4– 8 hours, so the tablets may be prescribed three times daily. The dose should be increased by one tablet every 2– 3 days until optimum effects are seen or adverse effects occur. If daily doses of <70 mg carbidopa are used, the peripheral decarboxylase will not be saturated and Mr LN will be more likely to suffer from nausea and vomiting. For this reason there are low-levodopa Sinemet preparations containing 25 mg carbidopa plus 100 mg levodopa. There are also preparations containing 10 mg carbidopa plus 100 mg levodopa that are suitable when higher levodopa doses are needed. This is not a problem with benserazide preparations. Most patients are initially controlled on 400– 800 mg levodopa daily.
  • 41. Mr LN is beginning to display symptomsof punding. This is charac- terised by repetitive pointless behaviours carried out for long periods of time at the expense of all other activities, and is associated with excessive dopaminergic stimulation. Punding and impulse control disorders, for example compulsive gambling, can be devastating and occur in up to 17% of patients treated with dopamine agonists. The risk is higher in younger patients with a history of addiction. Punding is managed by reducing and discontinuing dopaminergic stimulatory agents, predominantly dopamine agonists, although levodopa is also implicated. In view of Mr LN’s worsening motor function it is reasonable to reduce his dopamineagonist to manage
  • 42. the side-effect of punding, and to supplement this with (initially) low doses of levodopa in view of its superiority in improving motor function. Abrupt withdrawal of dopamine agonists has been associated with neuroleptic malignant syndrome and should therefore be avoided. As there are no defined reduction regimens for these agents, the rate of withdrawal can vary. In the case of Mr LN, complete withdrawal may not be necessary and so slow reduction would be appropriate, depending on symptom control. Reducing the daily dose by 3 mg at 5-day intervals would be a reasonable regimen. How can the adverse effects of levodopa be minimised? A10 The peripheral side-effects of levodopa are significantly reduced by combining it with a dopa-decarboxylase inhibitor
  • 43. (see A5); however, peripheral side-effects may still occur. Levodopa therapy is therefore best started at a low dose and increased gradually. In addition, ensuring that the drug is taken with or after food can reduce the incidence of nausea and vomiting still further. Using domperidone, a dopamine antagonist that does not cross the blood– brain barrier, can also reduce nausea and vomiting. Doses of 10– 20 mg domperidone 1 hour before levodopa preparations areeffective. Which of the long-term complications of levodopa therapy is Mr LN suffering from? A11 End-of-dose akinesia and dyskinesias.
  • 44. Initial treatment with levodopa leads to sustained improvement through- out the day. Most patients will show a slow improvement in response during the first 18– 24 months of treatment. Symptoms are then ade- quately controlled for 3– 5 years. Unfortunately, after long-term treat- ment (>5 years) only about 25% of patients continue to have a good, smooth response. The main complications that develop are fluctuations in response, dyskinesias, psychiatric side-effects and partial or substantial loss of efficacy. Fluctuations in response initially consist of end-of-dose deteriora- tion or end-of-dose akinesia. End-of-dose akinesia is the term used when the therapeutic effects of a dose of levodopa are lost. This
  • 45. commonly occurs first thing in the morning (after the longest dosage interval) or just before or after a dose during the day, when the effect of the previous dose wears off and before the next one is taken or becomes effective. Patients may become immobilised and unable to do anything except wait for the next dose. After prolonged treatment, gradual deterioration in symptoms may begin between 1 and 3 hours after a dose. Long-term levodopa therapy is also associated with the ‘on– off’ phenomenon. The patient develops a sudden loss of effectiveness (off), when ‘freezing’ occurs, which may last for only a minute or for up to several hours before normal function returns (on). Dyskinesias (abnormal movements) can occur in 60– 90% of patients
  • 46. and are usually dose-related. They are generally worse when the response to a dose is maximal (peak dose), and have been correlated with high levodopa plasma levels. They are therefore commonly seen after a dose has been taken. Symptoms include grimacing, gnawing and involuntary rhythmic jerking movements. What alterations to Mr LN’s therapy would you recommend in order to try to minimise these effects? A12 Increase the frequency of levodopa administration to 3- hourly and reduce the amount given with each dose. The most common approach to response fluctuation is to reduce each individual dose of levodopa and to increase the frequency of administration. This can reduce end-of-dose akinesia, but care must be
  • 47. taken to ensure a clinical response is maintained. End-of-dose bradykinesia or akinesia is thought to be caused by a progression of the underlying disease or an unexplained occurrence of symptoms of dopamine deficiency after an initial response to each dose. Although there is no change in the plasma half-life of levodopa, it appears that the pharmacological half-life is reduced. End-of-dose akinesia has been shown to be corrected by levodopa infusion; however, this is not a genuine therapeutic option, as levodopa must be infused in large volumes, owing to its acidity, and it also commonly causes thrombophlebitis. The initial approach taken to minimise the end-of-dose effect is therefore to try to reduce the dosage interval.
  • 48. Modified-release versions of co-careldopa (Sinemet) and co- beneldopa (Madopar) are available which result in a more prolonged and constant plasma level of levodopa. Both preparations lead to delayed action and lower peak concentrations than standard levodopa, but the risk of dose failure may be higher. Patients may still need to take controlled-release preparations every 3 or 4 hours, and may need doses of standard preparations to produce optimal clinical effects, especially with the first dose of the day. Dyskinesias may occur at the time of peak benefit, at the beginning or end of a dose, or both (diphasic), or during ‘off’ periods. A reduction in levodopa dose may reduce peak dose dyskinesias, as may the partial
  • 49. replacement of levodopa with a dopamine agonist. Diphasic dyskinesias may also be helped by partially replacing levodopa with dopamine agonists. ‘Off’ period dyskinesia may be improved by administering a fast- acting (e.g. dispersible) preparation or, for those occurring in the early morning, a controlled-release preparation last thing at night. Other measures to improve consistency of absorption should also be considered. This includes treating constipation (a common problem in Parkinson’s) and reviewing medications which can slow gastric emptying. Taking doses 1 hour before food (if tolerated) can also be helpful. High- protein meals may also reducelevodopa bioavailability because large neutral amino acids can compete with levodopa for absorption.
  • 50. How could you monitor and assess Mr LN’s response to these A13 changes? By charting his mobility regularly. Some neurology wards use mobility charts on which an indication of a patient’s mobility can be recorded at suitable intervals. Such charts can be a valuable aid to the manipulation of drug administration in order to optimise therapy. In addition to a score, a brief description of the patient’s condition may be added.
  • 51. Can you suggest any non-drug management that might benefitMr A14 LN? Mr LN may benefit from some remedial therapy, such as speech and language therapy, physiotherapy or occupational therapy. This should beavailable to patients with IPD and has been endorsed in NICE guidance for IPD. The role of these therapies is to maintain the maximum level of functional mobility and capacity to perform ADL. Early therapeutic intervention cannot reverse the course of Parkinson’s disease, but it can delay potential deformity and functional decline. Physiotherapy for Parkinson’s disease patients addresses the functional limitations caused by rigidity and bradykinesia. It may include an
  • 52. On No rigidity, mobilising Off Rigid, with or without tremor Unable to mobilise or only with assistance On with dyskinesia No rigidity, mobilising but with involuntary movements State Description exercise programme that focuses on maintaining flexibility, balance and strength. Scoring system for patients with PD chart
  • 53. Would Mr LN benefit from an antidepressant? A15 Probably. Depression is very common in Parkinson’s disease; approximately 40– 50% of patients suffer from depression at least once during the course of their disease. Depression in Parkinson’s disease is characterised by feelings of guilt, helplessness, remorse and sadness. It is independent of age, disease duration, severity of symptoms or cognitive impairment. Ensuring adequate treatment for Parkinson’s disease should be the first step before considering more specific antidepressant therapy. This has been achieved in Mr LN, so a trial of an antidepressant would be reasonable.
  • 54. If so, which would you choose? A16 A tricyclic antidepressant (TCA) or selective serotonin reuptake inhibitor (SSRI) could be prescribed for Mr LN. At present, there is insufficient evidence to recommend one antidepres- sant/antidepressant class over another. This lack of data is also highlighted in the NICE guidance. Clinical practice, as well as trial data, supports the use of some TCAs (e.g. amitriptyline, nortriptyline) in Parkinson’s dis- ease; however, they are often associated with anticholinergic effects and orthostatic hypotension, which may limit their usefulness. The SSRIs (e.g. fluoxetine, sertraline, citalopram)
  • 55. have also been shown to be effective in Parkinson’s disease. They are free of the anti- cholinergic effects associated with the tricyclics, and theoretical concerns that they may worsen parkinsonian symptoms have not been borne out by recent studies. Why has the consultant neurologist recommended entacapone, and does this necessitate the adjustment of Mr LN’s other IPD therapy? A17 Mr LN is still displaying signs of end-of-dose akinesia, despite appropriate adjustment to his levodopa therapy. Entacapone ther- apy may improve these symptoms, but his otherIPD therapies will need to be adjusted as the drug is introduced.
  • 56. Entacapone is a catechol-O-methyltransferase (COMT) inhibitor and works in synergy with Sinemet and Madopar preparations, resulting in a 30– 50% increase in levodopa half-life. Entacapone works by inhibiting the peripheral metabolism of levodopa by the enzyme COMT, thereby increasing its availability to the brain. Entacapone is licensed for use as an adjunct to levodopa in Parkinson’s disease. Studies have shown that its use can significantly reduce ‘off’ time and increase ‘on’ time in patients with ‘wearing-off’ episodes. It should be given at a dose of 200 mg with each dose of levodopa, up to 10 times daily. The introduction of entacapone to a regimen can cause a new onset or worsening of existing dyskinesias. In order to minimise this, the daily dose of levodopa should be reduced by about 10– 30% by extending
  • 57. the dosing intervals and/or by reducing the amount of levodopa per dose. In practice, the choice varies from patient to patient. Tolcapone, the only other commercially available COMT inhibitor, differs from entacapone in that it also blocks centralCOMT. Tolcapone was withdrawn in 1998 after a number of cases of hepatitis, but has since been relaunched with strict monitoring parameters and guidance onuse.
  • 58. What counselling points would you highlight to a patient A18 commencing entacapone? Entacapone can discolour the urine reddish brown. Patients should also be advised that nausea and vomiting can occur due to augmentation of levodopa, and that diarrhoea is a common adverse effect. The large size of the tablets maycause problems for patients with swallowing difficulties.
  • 59. In view of Mr LN’s concerns, what options are there for A19 rationalising his medications? The use of combination products and alternative formulations could be considered. In the literature it has been noted that approximately 20% of patients with IPD are non-adherent with their medication. Younger patients and those with complex regimens are associated with the highest levels of non-adherence. A number of pharmaceutical companies have recently developed combined preparations and modified-release preparations in an attempt to reduce the patient’s ‘pillburden’. Stalevo is a combined preparation of levodopa, carbidopa and
  • 60. entacapone formulated into one tablet. A Stalevo tablet is smaller than a tablet of entacapone alone andis thus also helpful in patients with swallowing difficulties who require therapy. Stalevo is available in a number of strengths, although dose titration is more limited than with the individual components. Mr LN could be started on Stalevo in place of Sinemet and entacapone. The transdermal patch rotigotine and the oral formulations ropini- role XL and pramipexole PR are designed for once-daily administration. Rotigotine is particularly useful where swallowing difficulties are trouble- some, or compliance with tablets is poor. There are therefore a number of options for the rationalisation of Mr LN’s medication regimen. A reasonable option would be to convert
  • 61. each dose of levodopa/carbidopa and entacapone to one tablet of Stalevo 100/25/200. Ropinirole could also be converted to the XL preparation, as recommended in the Summary of Product Characteristics, to one 8 mg tablet daily. These changes would more than halve the number of tablets taken by Mr LN from 16 per day to seven. Which drugs might have contributed to Mr LN’s symptoms? A20 All the drugs prescribed for Mr LN can cause the psychiatric complications described. Levodopa causes a variety of psychiatric symptoms, including hallucina- tions. Dopamine agonists such as ropinirole can cause
  • 62. central nervous system effects such as hallucinations and confusion. These psychiatric complications are especially common in elderly patients. In addition, progression of the disease itself may contribute to these symptoms. A21 What adjustments would you recommend be made to Mr LN’s medication? Gradually reduce his dose of ropinirole XL and stop ifnecessary. The psychiatric complications of most antiparkinsonian drugs are dose- related and often respond to a reduction in dosage. It is generally recommended to reduce or eliminate
  • 63. antiparkinsonian drugs in the following order, corresponding to their relative propensity to cause psychiatric problems versus degree of antiparkinsonian activity: anticholinergics, amantadine, MAOBI, dopamine agonist, levodopa. Mr LN’s dose of ropinirole should be reduced to the point of improv- ing his hallucinations without drastically worsening his parkinsonism, if possible. It should be reduced gradually, as sudden withdrawal of dopaminergic agents may precipitate a neuroleptic malignant syndrome. The levodopa dose should only be reduced if hallucinations persist after elimination of all other antiparkinsonian agents.
  • 64. What course of action would you suggest to improve Mr LN’s A22 symptoms? Add a low dose of an ‘atypical’ antipsychotic drug, or consider a cholinesterase inhibitor. Haloperidol and chlorpromazine are effective antipsychotics but are not recommended for Parkinson’s patients because of theircapacity to block striatal dopamine D2 receptors and exacerbate parkinsonism. The newer ‘atypical’ antipsychotics (e.g. clozapine, olanzapine, risperidone, quetiapine) are relatively free of D2-receptor-blocking potential, and in principle should improve psychotic features without worsening parkinsonism. The best studied of these is clozapine, which
  • 65. has been shown to reduce hallucinations without worsening parkinsonism. However, the potential for clozapine to cause agranulocytosis (1 – 2% of patients) and the consequent rigorous monitoring requirements often deter its use. Olanzapine and risperidone have proved less effective than clozapine in comparative studies, and have also worsened parkinsonism. The early work with quetiapine was more hopeful; however, more recent controlled studies have been negative. When quetiapine is considered, a starting dose of 12.5 mg at bedtime is recommended and the dose should be titrated upwards at 3– 5-day intervals until the desired effect is achieved. The cholinesterase inhibitors rivastigmine, galantamine and
  • 66. donepezil, which were developed primarily for Alzheimer’s disease, have been shown to reduce psychotic features and improve cognition in some patients with Parkinson’s disease dementias. Most studies have used rivastigmine and it is the only agent licensed for use in Parkinson’s disease dementias. A relative cholinergic excess is already present in the striatum of Parkinson’s disease patients, and thus the use of an agent likely to support this imbalance further should only be undertaken under specialist supervision.
  • 67. What is the long-term outlook for Mr LN? A23 It is likely that Mr LN’s condition will continue to deteriorate with time, and that he will experience more ‘off’ time and increasing dyskinesias. His cognitive function may also decline further. Amantadine has been used for the management of dyskinesias based on its antiglutamate activity; however, its side-effects (confusion, hallucina- tions) would preclude its use in Mr LN. Other side-effects of amantadine include ankle swelling and livedo reticularis. A small number of other therapies can be used at this stage in patients with Parkinson’s disease, but their risks and financial implications are considerable.
  • 68. The use of apomorphine, a potent dopamine agonist licensed for the treatment of refractory motor fluctuations (‘off’ periods) in IPD, is a possibility in patients who deteriorate despite maximum tolerated oral therapy. The drug cannot be given orally and must be administered subcutaneously. It may be given as a continuous subcutaneous infusion or as single injections. It causes severe nausea and vomiting, so 3 days’ pretreatment with domperidone (20 mg three times daily) is used to minimise this. Domperidone therapy may be continued until tolerance to this side-effect develops. Apomorphine is effective within 5– 10 minutes, and patients may remain in the ‘on’ state for up to 60 minutes. During this time an oral dose of medication should have taken effect. Many patients are
  • 69. helped by up to five injections a day, although up to 10 may be needed in some patients. If the number of injections needed ishigh, then continuous infusions of apomorphine should be considered. If the nausea and vomiting can be overcome, apomorphine therapy is generally well tolerated. Bruising, nodules or abscesses may form at the site of an infusion, so the site should be changed daily. As Mr LN has had psychotic features in the past that resolved following discontinuation of his dopamine agonist, the use of apomorphine may not be appropriate and may further exacerbate this symptom. The use of surgical techniques such as subthalamic deep brain stim- ulation (STN-DBS) may also be precluded in Mr LN by his previous psychotic symptomatology. The relatively high morbidity
  • 70. and mortality rate associated with lesioning operations such as thalamotomy and palli- dotomy has recently made STN-DBS more favourable. STN-DBS involves the placement of tiny wires into the subthalamic nucleus (STN). These emit continuous electrical impulses from a neurostimulator, which is sim- ilar to a heart pacemaker. This stimulation can have a positive effect on the brain activity involved in controlling movement, and can improve tremor, stiffness, slowness and dyskinesia. In addition, with improvement in these symptoms medication can be reduced, thereby further reducing dyskinesias. Mr LN may be an appropriate candidate for Duodopa, a gel formulation of levodopa/carbidopa that is infused directly into the duodenum. Duodopa is designed to mimic the pharmacokinetic profile
  • 71. of endogenous dopamine release. The technique, albeit initiated using a nasogastric tube, requires a percutaneous gastrostomy tube for long- term administration. Such a procedure in patients with advanced Parkinson’s disease can be hazardous. Although there have been many recent advances in the management of IPD, there is still no cure. Ongoing research is targeted at neuroprotection strategies: interventions to protect or rescue vulnerable dopaminergic neurones, and to slow down or stop disease progression. Gene therapy trials are also currently under way.